Uworld / NBME

Réussis tes devoirs et examens dès maintenant avec Quizwiz!

A 63-year-old man with long-standing hypertension has undergone multiple changes in his antihypertensive regimen over the years with only minimal improvement in his blood pressure. A CT study of his thoracolumbar spine is performed for evaluation of chronic back pain, and the study incidentally reveals a small right kidney. Angiography demonstrates atherosclerotic narrowing of the right renal artery. The left renal artery is intact. You conclude that the flow in the right-sided artery is decreased by a factor of 16 compared to the left. By what percentage has the radius of the lumen been reduced?

50%

A 68-year-old woman is brought to the emergency department due to worsening lethargy. Her family states that the patient has had headache and nausea for the past several days, and today she was confused and lethargic. Medical history is significant for a previous ischemic stroke with no residual neurologic deficit, seizure disorder, hypertension, type 2 diabetes mellitus, and bipolar disorder. Vital signs are within normal limits. On physical examination, the patient is somnolent and responds to painful stimuli only. Mucous membranes are moist and jugular venous pressure is normal. The lungs are clear to auscultation and heart sounds are normal. There is no extremity edema. Laboratory evaluation reveals serum sodium of 118 mEq/L; blood urea nitrogen and serum creatinine are within normal limits. Serum osmolality is low and urine osmolality is high. Which of the following medications is the most likely cause of this patient's condition? A.Canagliflozin B.Carbamazepine C.Furosemide D.Lithium E.Spironolactone

B This patient has symptomatic hyponatremia (eg, somnolence, lethargy), and her laboratory studies (eg, low serum osmolality, high urine osmolality) are consistent with the syndrome of inappropriate antidiuretic hormone secretion (SIADH). Antidiuretic hormone (ADH) secretion by the hypothalamus and posterior pituitary stimulates the renal collecting ducts to reabsorb water into the systemic circulation. This action lowers serum osmolality and sodium and increases extracellular volume. Patients with excessive ADH activity (ie, SIADH) typically have the following manifestations: Serum sodium and osmolality decrease, leading to hypotonic hyponatremia. Urinary water excretion decreases, increasing urine osmolality and creating a concentrated urine. Clinical euvolemia, which is reflected by an absence of edema, lung crackles, and jugular venous distention (signs of hypervolemia) along with absence of dry mucous membranes and elevated blood urea nitrogen (BUN) and creatinine (signs of hypovolemia). In this case, the patient's SIADH is likely due to carbamazepine, an antiepileptic drug that induces ADH production and increases renal sensitivity to ADH. Other medications associated with SIADH include antidepressants (eg, selective serotonin reuptake inhibitors and tricyclic antidepressants), anticancer drugs (eg, cyclophosphamide), certain antidiabetic drugs (eg, chlorpropamide), and drugs of abuse (eg, MDMA [ie, ecstasy]).

A 53-year-old woman comes to the clinic due to a skin rash. For the last 3 weeks, she has had a pruritic rash of worsening severity involving the posterior thighs. The patient recently began an exercise program to lose weight and has been applying a topical analgesic cream to her thighs and buttocks after her workouts. Her past medical history is unremarkable, and she does not use tobacco, alcohol, or illicit drugs. On physical examination, there is an erythematous rash with blisters, ulcers, and weeping drainage involving the posterior thighs bilaterally, as shown in the image below. Which of the following is the most likely finding on skin biopsy? A.Acanthosis B.Dyskeratosis C.Hypergranulosis D.Hyperkeratosis E.Spongiosis

E Eczematous dermatitis (eczema) is a group of conditions characterized by erythematous, papulovesicular, weeping lesions. This patient has acute allergic contact dermatitis (ACD), a form of eczema caused by a type IV (delayed) hypersensitivity reaction to an antigen on the skin surface. In ACD, antigens are taken up by antigen-presenting Langerhans cells and presented to CD4+ T cells in regional lymph nodes. The T cells are activated and migrate to the skin, where they incite an inflammatory response within 24 hours of antigen re-exposure. Acute eczematous dermatitis is characterized histologically by spongiosis, an accumulation of edema fluid in the intercellular spaces of the epidermis. The intercellular bridges become more distinctive in an edematous background, and the epidermis is often described as "spongy." Eventually, the edema can become so marked as to form intraepidermal vesicles. A perivascular infiltrate of lymphocytes and eosinophils may also be seen; this may involve only the superficial dermis (if due to exposure to a surface antigen) or extend to include the deeper vessels (if due to a systemic antigen exposure [eg, drugs]). With persistent antigen exposure, lesions may become less edematous and weepy. Thickening of the stratum spinosum (acanthosis) and stratum corneum (hyperkeratosis) produces raised, scaly plaques(Choices A and D).

A 12-year-old girl is brought to the clinic by her parents after she is found to have hypertension by her school nurse. The patient has no symptoms and reads a book during the office visit. The girl had not received routine well-child care until immigrating to the United States several months ago. She has had several episodes of fever and abdominal pain, which her parents have been able to treat with over-the-counter antibiotics. The patient's blood pressure is elevated on several readings in the office. There is no family history of hypertension. Renal ultrasound reveals dilated calyces with overlying cortical atrophy bilaterally, mostly in the upper and lower poles. Which of the following is the most likely cause of this patient's condition? A.Autosomal dominant polycystic kidney disease B.Malignant hypertension C.Multicystic dysplastic kidneys D.Posterior urethral valves E.Reflux nephropathy

E PUV is common in males This patient's history of recurrent fever and abdominal pain along with imaging findings consistent with renal scarring indicate recurrent pyelonephritis. Pyelonephritis results from retrograde flow of infected urine from the bladder into the ureter. Normally, the ureters travel through the bladder wall at an oblique angle. When the bladder fills, the intramural ureter becomes compressed. This flap-valve mechanism prevents retrograde flow of urine. However, this mechanism does not work correctly if the ureter enters the bladder wall at a more perpendicular angle, a condition known as vesicoureteral reflux (VUR). Patients with VUR are at much higher risk for chronic pyelonephritis. Inflammation can occur from pyelonephritis or from VUR itself due to hydrostatic pressure on the papillae. Ongoing injury leads to renal scarring, most commonly at the upper and lower poles of the kidney in which compound papillae are found. Compound papillae are always open, unlike simple papillae in the mid kidney, and are therefore much more susceptible to dilation and subsequent injury. If uncorrected, VUR can lead to loss of nephrons and secondary hypertension.

A 38-year-old man is evaluated for bloody stools, weight loss, and anemia. The patient's past medical history is unremarkable, and he does not take any medications. He uses tobacco and drinks alcohol "socially." The patient's father died of colon cancer at age 40. His younger sister suffers from endometrial cancer. Examination is remarkable for minimal right-sided abdominal tenderness without guarding to deep palpation. Stool is positive for occult blood. Colonoscopy shows a right-sided ulcerative colon mass. No polyps are noted. Biopsy of the mass reveals adenocarcinoma. A mutation in which of the following genes is most likely responsible for this patient's condition? A.APC B.BRCA1 C.MEN1 D.MSH2 E.VHL

This case is typical of hereditary nonpolyposis colon cancer (HNPCC), or Lynch syndrome, an autosomal dominant genetic predisposition to colon cancer. In patients with HNPCC, colon cancer occurs at a young age (age <50). Family history reveals a high incidence of colon cancer and, occasionally, extraintestinal (eg, endometrial) cancers in first-degree relatives. With HNPCC, there is an inherited mutation in one of the genes responsible for DNA mismatch repair (eg, MSH2, MLH1). The products of these genes proofread DNA during replication. Patients with HNPCC inherit a mutation in an allele, and mutation of the second allele occurs during adult life. When 2 dysfunctional copies are present, malignancies will develop readily. (Choice A) The APC gene is responsible for intercellular attachment. Its mutation leads to the formation of adenomatous polyps from normal mucosa. APC mutation is detected in most sporadic colon cancers and in all familial polyposis syndromes but not in HNPCC. (Choice B) Mutations in the tumor suppressor gene BRCA are classically associated with breast and ovarian cancers. (Choice C) MEN1 mutations are associated with multiple endocrine neoplasia type 1 syndrome (parathyroid, pituitary, and pancreatic adenomas).

A 70-year-old man comes to the emergency department due to severe right-sided chest pain and shortness of breath after a fall. While doing yard work, the patient fell off of a ladder, striking his back on several large stones. Immediately afterward, he felt sharp pain over his posterior chest and had trouble catching his breath. Blood pressure is 120/80 mm Hg, pulse is 88/min, and respirations are 24/min. Chest x-ray is shown in the exhibit. Physical examination of the right side of the chest would most likely reveal which of the following? A.Bronchophony B.Dullness to percussion C.Increased diaphragmatic excursion D.Prolonged expiration E.Subcutaneous crepitus F.Wheezing

This patient experienced sudden-onset shortness of breath and chest pain following chest trauma; chest x-ray shows several rib fractures and a collapsed right lung, findings consistent with traumatic pneumothorax. Traumatic pneumothorax can result from puncture of either the chest wall (eg, penetrating chest trauma) or the lung (eg, by fractured ribs), allowing air to enter the pleural space. The communication pathway causes intrapleural pressure to equalize with atmospheric pressure (ie, loss of intrapleural negative pressure), which prevents lung expansion during inspiration and leads to breathing difficulty. Because air in the pleural space insulates sounds and vibrations originating in the airways, physical examination reveals decreased breath sounds and decreased tactile fremitus on the affected side. In addition, the relatively low density of air compared to normal lung tissue causes hyperresonance to percussion. Disruption of the skin or parietal pleura during the initial injury often allows air to enter the subcutaneous tissues of the chest wall, which is detected as subcutaneous crepitus (ie, crackling during palpation).

A 5-year-old boy is brought to the office by his parents for a well-child visit. His mother says, "My son is doing very well. He is able to read at a first-grade level and knows his name and address. He loves to sing, dance, and play house with his older sisters. Everything seems fine with him; we just want to make sure his vaccinations are up to date before our family trip overseas." The physician administers the appropriate vaccinations. Just before leaving, the father says, "I think you should know that my son is playing with his sister's dolls and doesn't seem to like cars or trucks like most other boys. Is that normal?" Which of the following is the most appropriate response to the father's concerns? A."I hear your concern; let me reassure you that many boys play with dolls as a normal part of exploring the world." B."I understand your concern, but at his age it is best to allow him to play freely." C."I understand your concern; it may help to have him play with male friends and spend more time with you." D."It is helpful that you brought this up. Your son may have gender dysphoria and need additional support and assessment." E."There is no need to worry; children don't fully develop gender identity until adolescence."

A

A 35-year-old woman, gravida 1 para 0, comes to the office for an initial prenatal visit. The patient has had no vaginal bleeding or abdominal pain and has not yet felt fetal movement. Menarche occurred at age 13. She sometimes has heavy menstrual bleeding with passage of clots, and other times she skips her menses. The patient's estimated gestational age based on her last menstrual period is 16 weeks. She has type 1 diabetes mellitus that is controlled with insulin. She smokes a pack of cigarettes daily. As part of her prenatal laboratory screening, a second-trimester maternal serum quadruple screen is performed and reveals an elevated alpha-fetoprotein level. Which of the following is the most likely etiology of this patient's abnormal screening result? A.Dating error B.Down syndrome C.Edwards syndrome D.Fetal growth restriction E.Fetal heart defect F.Hydatidiform mole

A Clue in this q was the fact this lady has irregular menses and the answer choices are so weird and confusing they dont make sense because for it to be down and edwards AFP will be low FGR shows decreased estriol levels and not increased AFP hydatidform mole would have significantly increased HCG level and cardiac malformation wont cause an increase in AFP The maternal serum quadruple screen is used to identify pregnancies at risk for congenital defects or fetal aneuploidy (eg, Down syndrome). It measures the concentration of alpha-fetoprotein (AFP), estriol, β-hCG, and inhibin A in the maternal blood. AFP is a glycoprotein produced in the fetal liver and gastrointestinal tract. Levels are dependent on gestational age and maternal conditions such as diabetes mellitus (typically associated with decreased MSAFP and estriol levels). Therefore, accurate pregnancy dating and complete medical history are required to correctly interpret an AFP level. The most common cause of an abnormal AFP level is inaccurate pregnancy dating (ie, dating error). In patients with irregular menses, dating by a last menstrual period can underestimate the true gestational age. Therefore, these patients require a fetal ultrasound, which can accurately determine gestational age and evaluate for other common causes of elevated MSAFP levels, which include multiple gestation (eg, twin pregnancy), open neural tube defects, and abdominal wall defects.

A 1-month-old, full-term African-American boy is brought to the office for a routine check-up. He is exclusively breastfed. The boy does not have any medical problems, and his hospital stay after birth was unremarkable. His mother consumes a well-balanced diet, takes no medications, and does not drink alcohol. The boy is at the 50th percentile for weight and the 60th percentile for height. His physical examination is normal. At this time, supplementation with which of the following should be recommended for this infant? A.Folic acid B.Iron C.Riboflavin D.Thiamine E.Vitamin A F.Vitamin D

Breast milk is the gold standard of infant nutrition; it contains proteins, carbohydrates, fats, vitamins, trace minerals, immunoglobulins, amino acids, and enzymes. All vitamins and trace minerals are present in adequate amounts except vitamins D and K. Vitamin K is supplemented by an intramuscular injection at delivery to prevent hemorrhagic disease in the newborn. Regular sunlight exposure typically provides sufficient vitamin D. However, infants are generally shielded from direct sunlight due to sunburn risk. In addition, infants with dark skin pigmentation require more sunlight exposure to produce adequate vitamin D. Melanin is a natural sunblock and prevents ultraviolet rays from reaching the skin for vitamin D synthesis. Prolonged vitamin D insufficiency can lead to rickets (inadequate bone and cartilage mineralization). Therefore, all exclusively breastfed infants should receive vitamin D supplementation to prevent rickets. Formula-fed infants do not require supplementation as formula is fortified with adequate amounts of vitamin D.

A 50-year-old woman comes to the emergency department with severe upper abdominal pain. The pain started suddenly and is sharp and colicky. She has also vomited several times throughout the day, including once while in the emergency department. The patient describes several prior episodes of similar abdominal pain that resolved spontaneously without treatment. Her other medical problems include complicated appendicitis when she was 22 years old. Her temperature is 37.8 C (100 F) and pulse is 112/min. Abdominal examination shows cessation of inspiration with deep palpation of the right upper quadrant. Laboratory assessment shows a serum bilirubin of 0.8 mg/dL and a serum alkaline phosphatase of 100 U/L. Initial imaging studies are equivocal. Which of the following diagnostic test results would be most specific for acute cholecystitis? A.Distended duodenum on upper gastrointestinal series B.Echogenic structures inside the gallbladder on abdominal ultrasound C.Failed gallbladder visualization on radionuclide biliary scan D.Increased serum aspartate and alanine aminotransferase levels E.Opacities in the right subcostal area on abdominal x-ray

C Acute cholecystitis is most often caused by gallstones obstructing the cystic duct. The diagnosis can be made by identifying signs of gallbladder inflammation (eg, wall thickening, pericholecystic fluid) on ultrasonography. When ultrasound is inconclusive, nuclear medicine hepatobiliary scanning (ie, cholescintigraphy) can be used to assess cystic duct patency and make the diagnosis.

A 32-year-old man comes to the clinic due to several months of fatigue and weight loss. The patient has no significant medical history. He drinks 1 or 2 alcoholic beverages daily and has used illicit intravenous drugs in the past. He is sexually active with his girlfriend. The patient has a maternal aunt with hypothyroidism, but his family history is otherwise unremarkable. Temperature is 36.7 C (98 F), blood pressure is 110/70 mm Hg, pulse is 65/min, and respirations are 18/min. Physical examination is normal. A liver biopsy is obtained, and light microscopy reveals large hepatocytes filled with finely granular, homogeneous, pale pink cytoplasm. Which of the following is the most likely diagnosis in this patient? A.Alcoholic steatohepatitis B.Hemochromatosis C.Hepatitis B infection D.Hepatitis C infection E.Nonalcoholic steatohepatitis

C Hepatitis B infection, one of the most common causes of hepatic injury in the United States, is frequently transmitted sexually or via percutaneous inoculation (eg, intravenous drug use). A distinct histopathologic manifestation of chronic hepatitis B infection is the accumulation of hepatitis B surface antigen within infected hepatocytes. This results in the appearance of a finely granular, diffusely homogeneous, pale eosinophilic cytoplasm ("ground-glass" hepatocytes). Other nonspecific morphologic changes include hepatocyte necrosis (eg, ballooning degeneration), apoptosis, steatosis, and portal inflammation with mononuclear inclusions (lymphocytes, macrophages). Acidophil (Councilman) bodies, deeply eosinophilic globules that represent shrunken apoptotic hepatocytes, can be seen in a variety of liver diseases.

The patient is hospitalized, and appropriate treatment is begun. After discussing the underlying cause of his condition—immunosuppression due to HIV infection—the patient agrees to take antiretroviral therapy consistently. One of the medications in his treatment regimen leads to the production of immature, noninfectious virions containing large polyproteins. Which of the following viral processes is most likely directly inhibited by this agent? A.Expression of glycoproteins on the virus cell surface B.Integration of the viral DNA molecule into the host genome C.Production of functional viral-encoded enzymes D.Removal of viral template RNA from the RNA-DNA hybrid

C The HIV genome contains 3 major genes (env, gag, and pol) that are transcribed as polycistronic mRNA and translated into polyproteins in the endoplasmic reticulum. The gag and pol polyprotein products (gag-pol) are cleaved by HIV protease into individual HIV enzymes and structural proteins. Protease inhibitors inhibit HIV protease from cleaving the gag-pol polyproteins, which results in the formation of immature, noninfectious virions. Protease inhibitors are a common component of first-line HIV antiretroviral therapy and are denoted by medications that end in "navir" (eg, darunavir, atazanavir, ritonavir).

A 26-year-old woman comes to the office due to several weeks of pain and muffled hearing in the left ear, along with a sensation of jaw clicking when chewing food. The patient has had episodic headaches and left-sided facial pain for several years that is worse when moving her jaw. She also has been told that she grinds her teeth while asleep. On examination, the patient is unable to fully open her mouth and reports pain with passive movement of the jaw. The external auditory canal and tympanic membrane are normal. A disorder affecting pterygoid muscle contractility is suspected. Involvement of which of the following nerves is most likely contributing to this patient's ear symptoms? A.Cervical spinal nerves B.Facial nerve C.Mandibular nerve D.Maxillary nerve E.Vestibulocochlear nerve

C This patient's facial pain, headaches, jaw dysfunction, and ear pain/muffled hearing are characteristic of temporomandibular disorder (TMD). The underlying etiology of TMD is multifactorial but includes temporomandibular joint (TMJ) derangement (eg, jaw misalignment, trauma) and hypersensitivity of the mandibular nerve. The mandibular nerve (CN V3) is the largest branch of the trigeminal nerve and supplies sensation to the TMJ, floor of the mouth, anterior tongue, and the lower part of the face. It also innervates the muscles of mastication (eg, medial and lateral pterygoid, masseter, temporalis), tensor veli palatini, and the tensor tympani in the middle ear, which dampens loud sounds by tensing the tympanic membrane. Mechanical derangement of the TMJ can result in irritation and hypersensitivity of the mandibular nerve, leading to pathologic contraction of the pterygoid muscles (worsening the jaw pain and dysfunction) and tensor tympani (causing ear pain and muffled hearing despite a normal otoscopic examination).

A 64-year-old man is brought to the emergency department due to severe chest pain, diaphoresis, and shortness of breath. Symptoms began suddenly on awakening and have worsened over the past 2 hours. Blood pressure is 150/90 mm Hg and pulse is 102/min. Physical examination reveals an S4. ECG shows ST-segment depression and T-wave inversion in the lateral leads. Coronary angiography reveals a ruptured atherosclerotic plaque, with a thrombus in the left circumflex artery that causes near-total occlusion. Normal endothelial cells surrounding the lesion have released large amounts of a chemical substance that help decrease thrombus propagation by inhibiting platelet aggregation. Which of the following substances was most likely secreted by these endothelial cells? A.Hageman factor B.Kallikrein C.Prostacyclin D.Protein C E.Serotonin F.Thromboxane A2

C The formation of a platelet plug (primary hemostasis) is essential for preventing bleeding after damage to vascular endothelium; it occurs in 3 steps: Platelet adhesion takes place via von Willebrand factor acting as a connector that binds platelets to underlying collagen Platelets become activated and secrete multiple substances, including adenosine diphosphate, ionized calcium, and fibrinogen, from their alpha and delta (dense) granules. Thromboxane A2 (Choice F), a prostaglandin, is released and acts as a vasoconstrictor and potent stimulator of platelet aggregation. Adenosine diphosphate also stimulates platelet aggregation. Balance is required as excessive platelet plug formation can lead to a pathologic thrombus that restricts blood flow (eg, myocardial infarction). To oppose the functions of thromboxane A2, the endothelium secretes prostacyclin (prostaglandin I2), which is derived from arachidonic acid and synthesized from prostaglandin H2 by prostacyclin synthase. Once secreted, prostacyclin acts locally to inhibit platelet aggregation and adhesion to the vascular endothelium and to cause vasodilation. Nitric oxide aids in these functions as well. Atherosclerosis can impair the ability of endothelial cells to synthesize prostacyclin and nitric oxide, creating localized predisposition to excessive platelet thrombus formation. A synthetic prostacyclin, epoprostenol, is used in the treatment of pulmonary hypertension, peripheral vascular disease, and Raynaud syndrome.

A 34-year-old missionary in southern Asia is traveling to a remote village and becomes stranded when his bus breaks down. He has no access to water for 36 hours, during which his urine osmolality reaches 1100 mOsm/L. Urine concentration depends primarily on the serum level of vasopressin, which is regulated by the neurohypophysis in response to plasma osmolality and blood volume. Which of the following nephron segments responds to vasopressin by increasing absorption of a specific solute that is important for generating a high medullary concentration gradient? A.Cortical segment of the collecting duct B.Early distal tubule C.Medullary segment of the collecting duct D.Proximal tubule E.Thin ascending limb of the loop of Henle

C Vasopressin, or antidiuretic hormone (ADH), is the primary physiologic inhibitor of free water excretion. This hormone acts on 2 major receptors, V1 and V2. Stimulation of the V1 receptor causes vasoconstriction and increased prostaglandin release; stimulation of the V2 receptor results in an antidiuretic response. ADH is secreted in response to plasma hyperosmolality and, to a lesser extent, depletion of the effective circulating volume. Water deprivation initially increases plasma osmolality, resulting in increased ADH secretion. This causes the kidney to produce concentrated urine, which helps to counteract the rise in plasma osmolality by reducing urinary free water excretion. The medullary portion of the collecting duct is of particular importance in the production of maximally concentrated urine as the medullary interstitium is the region of highest osmolarity in the kidney. In the setting of high serum ADH levels, a large osmotic gradient drives the absorption of free water into the hypertonic medullary interstitium. As water leaves the tubular fluid, urea concentration greatly increases. ADH also increases the number of passive urea transporters in the inner medullary collecting duct, allowing a substantial fraction of the highly concentrated urea to diffuse down its concentration gradient into the medullary interstitium. When ADH levels are high, this urea resorption contributes up to 50% of total osmolarity of the medulla, further increasing the water-absorbing capacity of the nephron.

A 25-year-old woman, gravida 4, para 0, comes to the office for evaluation of recurrent pregnancy loss. She has had 4 second-trimester losses with the same partner. The patient has menstrual cycles approximately 28 days with light bleeding for 2-3 days. Her past medical history is significant for left renal agenesis. Family history is negative for recurrent pregnancy loss. Laboratory studies are negative for anticardiolipin antibodies and lupus anticoagulant. Results of the patient's hysterosalpingogram are shown exhibit. Based on her evaluation, a pelvic MRI is ordered and shows an abnormal contour to the uterine fundus. Failure of which of the following process is the most likely underlying mechanism of this patient's condition? A) Development of the paramesonephric duct B) Fusion of the mesonephric duct C) Involution of the paramesonep duct D) lateral fusion of the paramesonephric duct

D Female reproductive tract development involves lateral/vertical fusion and involution of the paramesonephric ducts (eg, müllerian ducts), which give rise to the fallopian tubes, uterus, cervix, and upper vagina. Development of the paramesonephric and mesonephric ducts is closely linked; therefore, uterine anomalies often coexist with renal anomalies (eg, unilateral renal agenesis). Failed lateral fusion of the paramesonephric ducts can result in various anomalies. Incomplete lateral fusion of the upper segments can result in a bicornuate uterus characterized by an indentation in the center of the fundus. Complete lack of fusion can lead to uterine didelphys (double uterus and cervix). Failed involution of the paramesonephric ducts can result in a longitudinal uterine septum. When a patient has difficulty conceiving or recurrent pregnancy loss, structural uterine anomalies can be screened by hysterosalpingogram (HSG), which involves contrast injection through the cervix into the uterus with a concurrent pelvic x-ray. This patient's HSG shows 2 unfused uterine horns with a central filling defect, which can represent a bicornuate uterus or a longitudinal uterine septum (Choice C). MRI can distinguish these 2 anomalies as a septate uterus has a normal outer uterine contour.

A 54-year-old man comes to the office due to episodic burning substernal chest pain. His pain increases with activity and improves with rest. The patient has a history of hypertension and hyperlipidemia. He has smoked a pack of cigarettes daily for the past 30 years. His blood pressure is 140/85 mm Hg and pulse is 76/min. Cardiac auscultation reveals a S4 heart sound. Lung and abdominal examinations are unremarkable. An ECG at rest shows left ventricular hypertrophy. A myocardial perfusion scan reveals inducible ischemia of the inferior surface of the heart. Which of the following coronary arteries is most likely occluded in this patient? A) Left anterior descending coronary artery B) Left circumflex coronary artery C) Left main coronary artery D) Right coronary artery E) Right marginal branch

D The right and left main coronary arteries arise directly from the root of the aorta and provide the blood supply to the heart. The left main coronary artery divides into the left anterior descending (LAD) and circumflex coronary arteries, which supply most of the anterior and left lateral surfaces of the heart. In 85%-90% of individuals, the right coronary artery gives rise to the posterior descending artery. Such patients are said to have right dominant coronary circulation. The posterior descending artery runs down the posterior interventricular groove and supplies the posterior one third of the interventricular septum and most of the inferior wall of the left ventricle. The inferior (diaphragmatic) surface of the heart is formed by the left ventricle (two thirds) and right ventricle (one third), which are separated by the posterior interventricular groove. Because most individuals have right dominant circulation, occlusion of the right coronary artery is most likely to cause inducible ischemia of the inferior surface of the heart.

A 38-year-old man comes to the office due to a 2-month history of an enlarging, localized swelling on the left foot. Tying his shoelaces too tight leads to localized pain and numbness in the third and fourth toes. Examination shows a firm, nontender, well-circumscribed lesion on the dorsum of the foot as shown in the exhibit. The lesion transilluminates when a penlight is applied to it. Without treatment, which of the following is most likely to occur in this patient? picture shows a ganglion cyst A) paralysis B) bloodstream infection C) Contained crystal aggregation D) Spontaneous resolution

D This patient has a ganglion cyst, an outpouching of connective tissue arising from tendon sheaths, joint structures (eg, joint capsule), or bursae. Clear mucinous or gelatinous fluid accumulates in the sac, giving rise to a round, well-circumscribed, firm cyst that transilluminates on penlight examination. Ganglion cysts are most common on the dorsal and volar wrists, but dorsal foot lesions are also common. Most ganglion cysts resolve spontaneously and do not require specific treatment. Cysts that raise cosmetic concerns or cause pain or paresthesia (due to nerve compression) can be treated with needle aspiration if desired. However, recurrence is common, and patients may eventually require surgical excisio

An 18-year-old man comes to the office due to abnormal skin pigmentation and premature hair graying. His nails are also thin and break easily, and he has white patches on his tongue. The patient's father had similar features at a young age and died of progressive pulmonary fibrosis. Physical examination shows diffuse gray hair, areas of reticular hyperpigmentation on the torso, dystrophic nails, and oral leukoplakia. Genetic testing reveals a loss-of-function mutation affecting the telomerase reverse transcriptase gene. Which of the following cell types is likely to be most affected by this mutation? A.Cardiac myocytes B.CNS neurons C.Compact bone osteocytes D.Hematopoietic stem cells E.Vascular endothelial cells

D This patient has dyskeratosis congenita; a genetic disorder involving a mutation in the genes related to telomere maintenance (eg, telomerase reverse transcriptase) that results in short telomeres. Telomeres are a complex of protein (eg, shelterin) and DNA repeats (eg, TTAGGG) at the ends of chromosomes that prevent chromosomal degradation and fusion with neighboring chromosomes. With each cellular division telomeres progressively shorten, eventually reaching a critical length at which apoptosis or senescence is triggered. In cells with a high turnover rate (eg, epithelial cells, lymphocytes, hematopoietic stem cells), telomere length is maintained by telomerase. This complex, composed of an RNA template and a reverse transcriptase, adds DNA repeats to the end of telomeres as they are lost with cell division. Without normal telomerase activity, rapidly dividing cells cannot maintain chromosomal integrity, triggering premature cell death. In patients with short telomere disorders, loss of these cells can cause characteristic mucocutaneous changes (eg, oral leukoplakia, dystrophic nails), bone marrow failure (eg, pancytopenia), and pulmonary fibrosis (due to alveolar epithelial dysfunction).

A 34-year-old man comes to the physician reporting one week of inability to extend his right wrist and several of his fingers on the same hand. He first began having difficulty while trying to type his thesis for graduate school on his computer. He is right-handed and does not recall any trauma. He has a history of asthma that was diagnosed while he was in college. Physical examination demonstrates impaired dorsiflexion of the right wrist with normal strength of the left wrist. Laboratory studies show: Complete blood count Hemoglobin 13 g/dL Platelets 320,000 /µL Leukocyte count 14,000 cells/µL Neutrophils 50% Eosinophils 28% Lymphocytes 17% Serum antibodies against neutrophil myeloperoxidase are positive. This patient most likely has which of the following conditions? A.Allergic bronchopulmonary aspergillosi B.Anti-glomerular basement membrane antibody disease C.Carpal tunnel syndrome D.Eosinophilic granulomatosis with polyangiitis E.Idiopathic pulmonary fibrosis F.Systemic sclerosis

D This patient most likely has eosinophilic granulomatosis with polyangiitis (Churg-Strauss). This small to medium vessel vasculitis is characterized by late-onset asthma, rhinosinusitis, and eosinophilia, though it can involve many other organ systems including the kidneys, gastrointestinal tract, and cardiovascular system. Asymmetric multifocal neuropathy (mononeuritis multiplex) is particularly common due to the vasculitis affecting the epineural vessels (eg, wrist drop due to radial nerve involvement). Other common manifestations include skin nodules, migratory/transient pulmonary infiltrates, and paranasal sinus abnormalities. In addition to peripheral eosinophilia, a frequent laboratory finding is antibodies against neutrophil myeloperoxidase, which most commonly have a pattern of perinuclear staining (p-ANCA).

A 72-year-old man comes to the primary care clinic with a 6-month history of back and bilateral thigh pain provoked by walking. He normally can walk only 2-3 blocks before having to stop due to pain. However, when he is out walking with his grandchild, leaning on the stroller seems to provide pain relief. The patient also notices an occasional tingling sensation in his lower extremities. He has no prior trauma or history of rheumatologic disorders. On physical examination, muscular strength is normal and the sensory findings are unremarkable. Peripheral pulses are full and symmetric. Thickening of which of the following ligaments is most likely contributing to this patient's current presentation? A.Anterior longitudinal B.Iliolumbar C.Interspinous D.Ligamentum flavum E.Sacroiliac

D This patient presents with spinal stenosis, an abnormal narrowing of the spinal canal occurring most commonly in the lumbar region. Compression of nerve roots results in a number of neurologic symptoms, including lower extremity pain, numbness/paresthesia, and weakness. The onset of pain with walking is referred to as "neurogenic claudication" as it may resemble symptoms seen in vascular claudication. However, the symptoms of spinal stenosis are posture-dependent. Extension of the lumbar spine (eg, standing, walking upright) further narrows the spinal canal and worsens the symptoms, whereas lumbar flexion (eg, walking uphill, leaning on a stroller/shopping cart) relieves the pain. Degenerative arthritis of the spine is the most common cause of spinal stenosis, typically occurring in patients age >60. Over time, the intervertebral disc degenerates and begins to protrude, resulting in a corresponding loss in disc height. This loss of height places a disproportionate load on the posterior aspect of the spinal column, leading to formation of facet joint osteophytes and hypertrophy of the ligamentum flavum (a strong elastic ligament supporting the posterior aspect of the spinal canal). These physiologic changes lead to mechanical compression of nerve roots and corresponding neurologic symptoms.

A 43-year-old man comes to the office due to fatigue and worsening pedal edema over the past several weeks. The patient was diagnosed with advanced HIV infection and started on antiretroviral therapy 3 months ago. He also has a history of hypertension. Physical examination shows bilateral lower extremity pitting edema. Serum creatinine is elevated and serum phosphorus is decreased compared to test results 3 months ago. Urinalysis reveals moderate proteinuria and glucosuria. Renal biopsy shows cytoplasmic vacuolization in the proximal tubules accompanied by loss of brush border and basement membrane denudation. Intracytoplasmic eosinophilic inclusions are also seen in the proximal tubules. There are no significant interstitial inflammatory infiltrates, and the glomeruli appear normal. Which of the following is the most likely diagnosis in this patient? A) HIV associated nephropathy B) hypertensive nephrosclerosis C) post infectious glomerunephritis D) tenofovir induced nephrotoxicity E) TMP-SMX induced nephrotoxicity

D This patient with HIV was recently started on antiretroviral therapy and subsequently developed evidence of proximal tubule dysfunction (eg, hypophosphatemia, glucosuria, proteinuria, biopsy abnormalities), raising concern for tenofovir-induced nephrotoxicity. Tenofovir is a common part of first-line HIV therapy. It is an adenosine analogue that is a member of the nucleotide reverse transcriptase inhibitors (NRTIs). NRTIs prevent reverse transcriptase from converting the HIV RNA genome into complementary double-stranded DNA, which inhibits viral replication. Tenofovir is primarily eliminated via the proximal tubule cells of the kidney. High concentrations of tenofovir in these cells can interfere with mitochondrial DNA synthesis and result in cellular damage. This typically manifests with acute kidney injury (eg, elevated creatinine, water retention) and/or signs of proximal tubule dysfunction (eg, phosphaturia, glucosuria, proteinuria). Histopathology may show damage to proximal tubule cells (eg, loss of brush border, basement membrane denudation) and evidence of giant mitochondria (eg, large eosinophilic inclusions). Renal abnormalities typically improve once tenofovir is discontinued.

A 35-year-old woman comes to the office for evaluation of anxiety. The patient works as a mechanical engineer and recently declined a promotion to be a project manager. Although her salary would have increased significantly, she felt that overseeing other employees and leading team meetings would be too stressful. She says, "If something were to go wrong, everyone would blame me. I don't even know why they offered me this position." The patient enjoys her job but tends to eat lunch by herself because she feels that her coworkers do not like her appearance or sense of humor. She has never had a long-term romantic partner but fantasizes about getting married someday. The patient lives alone, rarely socializes with friends, and is close with her mother, whom she describes as "the person I can always rely on no matter what." Which of the following is the most likely diagnosis? A) Acute stress disorder B) Adjustment disorder with anxiety C) Antisocial personality disorder D) Avoidant personality disorder E) Generalized anxiety disorder F)Paranoid personality disorder G) Schizoid personality disorder

D This patient's pattern of avoiding social interactions and hypersensitivity to criticism is characteristic of avoidant personality disorder. Patients with this disorder typically have very limited social relationships due to the fear of being judged, embarrassed, or rejected. They desire social acceptance and may fantasize about having relationships, as in this patient; however, they struggle with feelings of inadequacy and pursue relationships only if certain of absolute acceptance by others. Occupational dysfunction due to difficulties interacting with coworkers or turning down promotions due to fear of criticism is common.

A 66-year-old man comes to the emergency department due to dizziness, dysarthria, and bilateral limb ataxia. His symptoms have been steadily worsening over the last 2 weeks. He has no other medical problems and takes no medications. He has a 50-pack-year smoking history and does not use alcohol or illicit drugs. Chest x-ray shows a mass in his right lung. He is admitted to the hospital for evaluation, but his neurologic symptoms continue to progress. He dies several months later despite receiving appropriate treatment. Autopsy shows extensive cerebellar Purkinje cell degeneration. Which of the following best describes the etiology of this patient's neurologic condition? A) vascular B) infection C) endocrine D) Autoimmune

D this is a paraneoplastic syndrome often associated with small cell lung carcinoma This patient's lung mass, cerebellar symptoms, and autopsy findings are consistent with paraneoplastic cerebellar degeneration. This paraneoplastic syndrome is most commonly associated with small cell lung cancer as well as breast, ovarian, and uterine malignancies. Patients exhibit progressively worsening dizziness, limb and truncal ataxia, dysarthria, and visual disturbances (eg, diplopia, oscillopsia). Paraneoplastic cerebellar degeneration is due to an immune response against tumor cells that cross-reacts with Purkinje neuron antigens, leading to acute-onset rapid degeneration of the cerebellum. Anti-Yo, anti-P/Q, and anti-Hu are the most common antibodies detected in the serum. Antibodies are not always detectable; thus, paraneoplastic cerebellar degeneration is diagnosed clinically after excluding other conditions.

A 25-year-old man with an insignificant medical history was playing soccer when he suddenly collapsed. Despite all attempts to save his life, he died. Autopsy is performed. Gross examination of the heart reveals a weight of 580 g (normal: <350). The cut surface of the heart is shown in the image. If this patient had a preparticipation sports screening, cardiac auscultation would have likely revealed a murmur that increases in intensity after which of the following? A.Passive leg raising B.Phenylephrine infusion C.Squatting D.Sudden standing E.Sustained hand grip

D anything that decreases preload would intensify the murmur

A 1-hour-old boy is in the neonatal intensive care unit with tachypnea and hypoxia. The infant was born at 39 weeks gestation via cesarean delivery due to variable decelerations. The pregnancy was complicated by a lack of prenatal care. The infant weighs 3.2 kg (7 lb 1 oz). Physical examination shows a flattened nose and bilateral club feet. Breath sounds are markedly diminished bilaterally. The infant is intubated and mechanically ventilated, but his oxygen levels do not improve. The infant dies 1 hour later. Which of the following is most likely to be found during autopsy?

DX? potter syndrome; renal agenesis

A 47-year-old man comes to the office for follow-up of his elevated blood pressure. The patient's medical history is significant for stable angina pectoris, for which he takes sublingual nitroglycerin as needed. He has no orthopnea, paroxysmal nocturnal dyspnea, or lower extremity swelling. His blood pressure is 154/107 mm Hg and pulse is 86/min. Physical examination shows no abnormalities. The patient is started on a low dose of atenolol. Which of the following cellular changes will most likely occur as a direct effect of this medication? Camp in cardiomyocytes, JG cells and vasculator

Decrease in heart and kidney no change in vasculator Beta blockers are the preferred treatment for patients with chronic stable angina and hypertension. They relieve anginal symptoms by inhibiting sympathetic stimulation of the heart, reducing both heart rate and contractility. Because beta blockers reduce heart rate and blood pressure during rest and exercise, the onset of angina is avoided or delayed. At low doses, atenolol is a selective β1 adrenergic antagonist. β1 receptors are found in cardiac tissue and on renal juxtaglomerular cells, but not in vascular smooth muscle. The β1 receptor is a G protein-coupled receptor (GPCR) associated with Gs, which increases intracellular cAMP levels. Blockade of the β1 receptor leads to decreased cAMP levels in cardiac and renal tissue without significantly affecting cAMP levels in vascular smooth muscle (Choices C and D). GPCRs comprise a large family of 7 transmembrane domain receptors that activate intracellular second messenger systems. There are 2 principal second messenger systems involving GPCRs: cAMP signaling pathway: The activated Gs α subunit activates the enzyme adenylyl cyclase, which catalyzes the conversion of ATP into cAMP. Elevated cAMP leads to the activation of protein kinase A. Conversely, the Gi α subunit inhibits adenylyl cyclase, thereby reducing cAMP levels. Phosphatidylinositol signaling pathway: The activated Gq α subunit activates phospholipase C, which degrades membrane lipids into diacylglycerol (DAG) and inositol triphosphate (IP3). Protein kinase C is activated by DAG, and calcium is released from the sarcoplasmic reticulum under the influence of IP3.

A 34-year-old man comes to the office with his girlfriend for follow-up of type 1 diabetes mellitus. He was diagnosed 15 years ago and is currently treated with a combination of basal and rapid-acting insulin injections. The patient has been experiencing intermittent episodes of hypoglycemia with a blood glucose of 35-54 mg/dL. Two weeks ago, he was taken to the emergency department after passing out due to low glucose levels. The patient is prescribed an injectable medication to be administered at home by a caregiver when hypoglycemia is associated with impaired consciousness. This medication rapidly improves hypoglycemia by which of the following mechanisms? A.Decreasing uptake of glucose by fat and muscles B.Decreasing uptake of glucose by the liver C.Increasing gastrointestinal absorption of glucose D.Increasing gluconeogenesis E.Increasing glycogenolysis

E Hypoglycemia initially manifests with neurogenic (autonomic) symptoms, including anxiety, tremor, and sweating. However, as hypoglycemia worsens patients may develop life-threatening neuroglycopenic symptoms, including confusion, loss of consciousness, and seizures. Mild to moderate hypoglycemia is usually self-treated with oral fast-acting carbohydrates (eg, glucose tablets, fruit juices). However, when hypoglycemia becomes severe enough to induce unconsciousness, parenteral treatment is necessary. In a medical setting, severe hypoglycemia is usually treated with an infusion of glucose, but this requires intravenous access. In a nonmedical setting, severe hypoglycemia can be treated with emergency glucagon kits, which are produced in intranasal and subcutaneous/intramuscular formulations and can be administered by a caregiver or informed bystander. Glucagon rapidly corrects hypoglycemia by increasing hepatic glycogenolysis; the free glucose residues are then released into the blood, causing a return to consciousness within 10-15 minutes of administration. Subsequently, glucagon induces gluconeogenesis and suppresses hepatic glucose uptake, which provide a delayed but somewhat more sustained effec

A 26-year-old man comes to the office due to myalgia, nasal congestion, and cough. Temperature is 38.3 C (100.9 F). Examination shows nasal and pharyngeal hyperemia. Rapid influenza antigen testing of nasopharyngeal secretions is positive. Within the infected cells of the patient's respiratory tract, viral proteins are degraded and attached to major histocompatibility complex class I molecules, which are then expressed on the cell surface for presentation to cytotoxic CD8+ lymphocytes. Which of the following enzymes is most essential to initiating this process? A.Acid phosphatase B.Caspase C.Guanylate cyclase D.Myeloperoxidase E.Ubiquitin ligase

E The cellular immune response to a viral pathogen (eg, influenza) requires the presentation of viral antigens to effector immune cells (eg, CD8+ lymphocytes). The ubiquitin proteasome pathway (UPP) is essential to this process because of its role in breaking down native and foreign intracellular proteins. Ubiquitin ligase initiates the UPP by recognizing specific protein substrates and catalyzing the attachment of ubiquitin. The proteins tagged with ubiquitin are then broken down by a proteasome to peptide fragments which can be recycled into amino acids. The peptide fragments produced during this process can also be coupled to major histocompatibility complex (MHC) class I molecules in the endoplasmic reticulum and then presented on the cell surface. Cytotoxic CD8+ lymphocytes recognize nonnative (eg, viral) proteins on infected cells and trigger apoptosis of the cell by activating the caspase cascade through the release perforin and granzyme

A 43-year-old woman says she cannot sleep due to pain and tingling affecting the first three digits of both hands. She has no problems with her legs. The patient suffers from chronic renal failure due to uncontrolled hypertension and receives hemodialysis. On examination, she has bilaterally diminished sensation over the thumb, index and middle fingers, and the radial half of her ring finger. Which of the following is the most likely mechanism of this patient's condition? A.Endomysial inflammatory infiltration B.Endoneural arteriole hyalinization C.Endoneural multifocal demyelination D.Mutations of a muscle structural protein gene E.Nerve compression within an anatomic compartment

E arpal tunnel syndrome (CTS) is a peripheral mononeuropathy caused by compression of the median nerve in the carpal tunnel. The carpal tunnel is an area located on the palmar surface of the wrist (between the carpal bones and transverse carpal ligament) in which the median nerve and tendons of the long flexors of the digits pass through. There is little room for expansion in this anatomical compartment. CTS is associated with conditions that reduce carpal tunnel space, including pregnancy (fluid accumulation), hypothyroidism (glycosaminoglycan buildup), diabetes mellitus (connective tissue thickening), and rheumatoid arthritis (tendon inflammation). Patients with a long history of hemodialysis can also develop median nerve compression through deposition of β2-microglobulin (dialysis-associated amyloidosis) in the carpal tunnel. Symptoms and signs of CTS often occur bilaterally and include: Sensory impairment, pain, and/or paresthesias in the median nerve distribution (palmar surface of the first 3 digits and radial half of the 4th digit). Symptoms are usually worse at night. Motor weakness during thumb abduction/opposition and thenar atrophy. Tinel sign (tapping over the flexor surface of the wrist reproduces symptoms) and Phalen sign (flexion of the wrist reproduces symptoms).

A 27-year-old man comes to the office due to recurrent episodes of muscle weakness. He has no other significant past medical history. The patient's weight has been stable for the past few years, and his current BMI is 23 kg/m2. His blood pressure is 190/110 mm Hg supine and 195/110 mm Hg standing. His heart rate is 70/min supine and 72/min standing. The rest of the physical examination is unremarkable. Laboratory evaluation shows very low plasma renin activity. Overactivity of which of the following structures is most likely responsible for this patient's symptoms? A. Chromaffin cells of the adrenals B. Extra-adrenal paraganglion cells C. Juxtaglomerular cells of the kidney D. Zona fasciculata of the adrenals E. Zona glomerulosa of the adrenals F. Zona reticularis of the adrenals

E primary aldostronism This patient has features consistent with primary hyperaldosteronism (PH), a very common cause of secondary hypertension. PH is usually due to increased secretion of mineralocorticoids from bilateral nodular hyperplasia of the adrenal zona glomerulosa or an aldosterone-producing adrenal adenoma (Conn syndrome). The main effect of aldosterone is to stimulate absorption of sodium and excretion of potassium and hydrogen ions in the renal collecting tubules. Aldosterone secretion from the zona glomerulosa is normally regulated by angiotensin II and potassium levels. Overproduction of aldosterone can result in sodium retention, hypertension, and feedback suppression of the renin-angiotensin system (ie, very low renin activity). Some patients also develop metabolic alkalosis and hypokalemia, which can be exacerbated by increased distal tubule sodium delivery (eg, diuretics, increased sodium intake). Symptomatic hypokalemia may cause muscle weakness, cramps, and, occasionally, rhabdomyolysis and cardiac arrhythmias. Despite the increase in sodium absorption, hypernatremia and pedal edema are rarely observed in PH due to the phenomenon of aldosterone escape. The high aldosterone levels lead to increased intravascular volume and therefore cause increased renal blood flow (with resulting pressure natriuresis) and augmented release of atrial natriuretic peptide. This ultimately results in increased sodium excretion by the renal tubules, which limits net sodium retention and prevents the development of overt volume overload and significant hypernatremia.

A 32-year-old man comes to the office due to an acute febrile illness that developed after a deer-hunting trip in Arkansas. The patient recalls getting tick bites in the woods but initially had no symptoms. He began experiencing fever, chills, malaise, headache, and myalgia a week after his return. Physical examination shows a diffuse, maculopapular rash; clear oropharynx; normal lung and heart sounds; and mild, diffuse abdominal tenderness. Laboratory testing reveals leukopenia, thrombocytopenia, and elevated aminotransferase levels. Microscopic evaluation of the blood demonstrates mulberry-shaped, intracytoplasmic inclusions in monocytes. Which of the following is the most likely pathogen responsible for this patient's condition?

Ehrlichia chaffeensis is harbored by white-tailed deer in the southwestern, southeastern, and mid-Atlantic United States. It is transmitted to humans by tick bite (eg, lone star tick) and subsequently spreads to tissue rich in mononuclear cells (eg, bone marrow, lymph nodes, liver, spleen), leading to nonspecific symptoms (eg, fever, chills, myalgia, headache), maculopapular rash, and significant laboratory abnormalities (eg, lymphopenia, thrombocytopenia, elevated aminotransferases). E chaffeensis replicates in membrane-bound vacuoles within the cytoplasm of monocytes and can be visualized on peripheral blood or buffy coat examination as mulberry-shaped, intraleukocytic inclusions (morulae). Treatment with antibiotics (eg, doxycycline) is usually curative.

A 27-year-old man is brought to the emergency department by his wife. She says that he has been "acting crazy" for the last 2 weeks. He has hardly slept for the past 7 days and instead has worked on miscellaneous projects around the house. The patient spent several thousand dollars on new power tools to accomplish these tasks. When questioned, his speech is rapid and frenzied. He feels "spectacular" and is creating an "architectural masterpiece." The patient has had 2 previous depressive episodes. Which of the following medications is the most appropriate agent for long-term management of this patient? A.Bupropion B.Chlorpromazine C.Haloperidol D.Lorazepam E.Paroxetine F.Valproate

F This patient's euphoric mood, decreased need for sleep, hyperactivity, grandiosity, and pressured speech lasting more than a week are characteristic of an acute manic episode of bipolar disorder. Bipolar disorder is a highly recurrent illness requiring maintenance treatment with mood-stabilizing medications to decrease the risk of recurrent mood episodes. Preferred medications for bipolar maintenance treatment include lithium, the anticonvulsants valproate and lamotrigine, and the second-generation antipsychotic quetiapine.

A 53-year-old man comes to the emergency department due to severe lower back pain. The patient's back pain started 2 weeks ago without any preceding trauma and has been keeping him awake at night. The pain has acutely worsened over the past 2 days and is not relieved with over-the-counter pain medications. His vital signs are within normal limits. Examination shows low back pain that is worse with back flexion and raising of the legs; it radiates into his left leg. Pinprick in the perianal area does not cause rapid contraction of the anal sphincter. The rest of the neurologic examination is normal. Which of the following nerve roots is most likely to be involved in this patient's condition? A) T12 B)L2 C)L4 D)L5 E)S1 F)S4

F This patient's worsening low back pain with a positive straight leg test is consistent with subacute lumbosacral radiculopathy, which most often occurs due to a herniated disk; less common causes include neoplasm, fracture, or epidural hematoma/abscess. Compression of the spinal cord and/or nerve roots can cause a wide range of deficits based on the level(s) involved. This patient's perianal anesthesia with loss of the anocutaneous reflex indicates involvement of the S2-S4nerve roots. In an adult, the spinal cord terminates in a tapering fashion as the conus medullaris (T12-S4) at the L1-L2 vertebral level. The collection of spinal nerves below this point (eg, L3-S4) exit inferiorly through their respective intervertebral foramina and are referred to as the cauda equina (ie, horse's tail). Lesions at the L1-L5 vertebral levels can affect the conus medullaris and/or T12-S4 nerve roots, resulting in a characteristic constellation of symptoms: Radicular low back pain (ie, radiates down one or both legs) Saddle/perianal anesthesia (S2-S4) Bowel and bladder dysfunction Lower extremity weakness This pattern of neurologic deficits can occur with either conus medullaris syndrome (lesions around L1-L2 vertebrae) or cauda equina syndrome (lesions below L2 vertebra). Conus medullaris syndrome typically causes bilateral, symmetric lower extremity weakness with upper motor signs (eg, spasticity, hyperreflexia), while cauda equina syndrome tends to cause flaccid, asymmetrical lower extremity weakness. Due to the potential for further neurologic deterioration, affected patients require urgent evaluation including MRI of the spine.

A 34-year-old man comes to the office for a follow-up visit. The patient has had type 1 diabetes since age 12 and is diligent in taking insulin to achieve good glycemic control. Over the past few months, he has had more frequent episodes of hypoglycemia despite no changes in his treatment, diet, or physical activity, but the episodes have been associated with less severe hypoglycemic symptoms compared to prior episodes. Physical examination is unremarkable. Laboratory results show normal basic metabolic panel and serum TSH level. Hemoglobin A1c is 7.0%. Which of the following factors is primarily responsible for this patient's increased frequency of hypoglycemia? A.Decreased epinephrine release B.Decreased glucagon release C.Decreased growth hormone release D.Downregulation of insulin receptors E.Upregulation of beta-adrenergic receptors

Hints in the question: The patient has less severe hypoglycemic sympotoms ( recall the hypoglycemic sympotoms are beta adrenergic mediated this means this patient has down reg of Beta receptors choice E is knocked out) Also he has type 1 diabetes and is on exogenous insulin so theres no feedback the body will keep absorbing insulin even if the glucose levels are low. Epinephrine would be increased to give symptoms but it doesnt play as big of a role and downreg of insulin receptor would cause hyperglycemia. hence making B the most appropriate answer. recall this is an AI disease so as the disease progresses there is destruction of alpha cells too leading to decrease in glucagon too. uworld explanation As blood glucose levels fall through the physiologic range, insulin secretion by pancreatic beta cells decreases, leading to reduced peripheral uptake and utilization of glucose. At the same time, alpha cells increase secretion of glucagon, which is the primary counterregulatory hormone to insulin and acts on the liver to rapidly increase glycogenolysis and gluconeogenesis and decrease glycolysis. Patients with type 1 diabetes mellitus are at increased risk of hypoglycemia because exogenous insulin is not subject to the same regulation as endogenous insulin and will continue to be absorbed from the injection site despite falling glucose levels. Those with long-standing diabetes (ie, >5 years) frequently also have alpha cell failure with decreased glucagon secretion and therefore have an even greater risk of rapid hypoglycemia. Patients with pancreatogenic diabetes (eg, secondary to chronic pancreatitis) also typically have inadequate alpha cell reserve and can experience frequent hypoglycemia.

A 25-year-old previously healthy man comes to the office due to penile lesions. The patient first noticed a nontender papule on his penis a month ago, and it has progressively increased in size and number. He reports no fever, dysuria, or penile discharge. The patient is sexually active with multiple partners and uses condoms inconsistently. Physical examination reveals 3 soft, flesh-colored, papillary lesions measuring <1 cm on the dorsum of the penis. There are no enlarged inguinal lymph nodes, and the remainder of the examination is normal. Treatment is started with a topical agent that acts as a toll-like receptor agonist to increase antiviral cytokine production. Which of the following medications was most likely prescribed to this patient? A. Calcipotriene (10%) B. Imiquimod (51%) C. Pimecrolimus (26%) D. Salicylic acid (5%) E. Silver nitrate (6%) A) calciproteine B)Imiquimod C) Pimercrolimus D)Silver nitrate E) Salicylic acid

Imiquimod is a topical immunomodulatory drug that is used to treat many common dermatologic disorders associated with abnormal cell proliferation, including anogenital warts (ie, human papillomavirus infection), superficial basal cell carcinoma, and actinic keratosis. The antiviral and antiproliferative effects of imiquimod are primarily mediated through activation of toll-like receptor 7, which upregulates the proinflammatory transcription factor nuclear factor-kappa B (NF-κB). NF-κB increases transcription of proinflammatory genes, activating antigen-presenting cells (eg, Langerhans cells in the skin) and initiating an immune response involving natural killer cells, cytotoxic T cells, and type 1 helper T cells. This results in increased cytokine production (eg, IL-1, IL-12, interferon-alfa/gamma, tumor necrosis factor-alpha) and enhanced immune-mediated killing of aberrant cells (eg, cancer cells, virus-infected cells). Other antiproliferative effects of imiquimod include: Induction of apoptosis of aberrant cells through caspase activation via inhibition of BCL-2 Inhibition of angiogenesis by downregulating proangiogenic factors (eg, fibroblast growth factor) and upregulating angiogenesis inhibitors (eg, interferon-gamma, IL-12)

A 71-year-old man with chronic obstructive pulmonary disease (COPD) is hospitalized due to shortness of breath and wheezing. The patient is treated with bronchodilators and systemic corticosteroids for acute COPD exacerbation. His symptoms improve over the next 2 days. Before being discharged, an intramuscular vaccine containing inactivated viral components, which he normally receives annually, is administered. Several weeks later, the patient comes in close contact with an individual infected with the same virus contained in the vaccine. Which of the following is most likely to occur in this patient as a result of the vaccination?

Inactivated versions of the influenza vaccine stimulate the formation of neutralizing antibodies against the hemagglutinin antigen of included strains. Subsequent exposure to a strain of influenza included in the vaccine will not result in infection because the antibodies bind to hemagglutinin, thereby preventing hemagglutinin from attaching to the sialic acid receptor on host respiratory epithelial cells (preventing viral entry).

A group of investigators is studying the regulation of catecholamine synthesis in response to severe stress. In the experiments, subject rats are randomly assigned to either an experimental or a control group. The experimental rats undergo resection of the pituitary gland, and the control rats undergo craniotomy without pituitary resection. The experimental animals are subsequently found to have decreased production of epinephrine by the adrenal medulla and cortisol from the adrenal cortex compared with the control animals. Decreased activity of which of the following enzymes is most likely responsible for the lower epinephrine in the experimental animals?

PNMT The 3 main circulating catecholamines are dopamine, norepinephrine, and epinephrine. Norepinephrine and dopamine are produced in the central as well as the peripheral nervous system, whereas epinephrine is predominantly produced in the adrenal medulla. The first step in the synthesis of catecholamines is the conversion of tyrosine to dihydroxyphenylalanine (DOPA) by tyrosine hydroxylase. This is the rate-limiting step in the synthesis of catecholamines. DOPA is converted to dopamine by dopa decarboxylase (Choice B), which is then converted to norepinephrine by dopamine beta-hydroxylase (Choice C). In the adrenal medulla, norepinephrine is rapidly converted to epinephrine by phenylethanolamine-N-methyltransferase (PNMT). Expression of PNMT in the adrenal medulla is upregulated by cortisol. Because the venous drainage of the adrenal cortex passes through the adrenal medulla, cortisol concentrations in the medulla can be very high, and PNMT is expressed at a high level. However, following pituitary resection, the loss of ACTH leads to decreased synthesis of cortisol in the adrenal cortex. The result is decreased PNMT activity and reduced conversion of norepinephrine to epinephrine.

A 62-year-old man comes to the office due to worsening fatigue. The patient has long-standing type 2 diabetes mellitus complicated by nephropathy. Laboratory results are as follows: Hemoglobin9.4 g/dL Mean corpuscular volume 90µm3 Platelets200,000/mm3 Leukocytes7,500/mm3 Serum creatinine3.2 mg/dL Total bilirubin0.8 mg/dLSerum ferritin350 ng/mL Transferrin saturation30% (normal: 20%-50%) After appropriate treatment is initiated for his anemia, which of the following changes is most likely to occur in this patient's erythroid cells? A) Decrease in portoporphyrin levels B) increase in erythroid progenitor cells C) Decrease in reticuloendothelial destruction

Patients with chronic kidney disease (CKD) often develop normocytic anemia as the glomerular filtration rate worsens due to inadequate production of erythropoietin (EPO), a glycoprotein hormone produced by peritubular fibroblast cells in the renal cortex. EPO is released into the bloodstream in response to renal tissue hypoxia and subsequently acts on erythrocyte progenitor cells (erythroid colony-forming unit cells) in the bone marrow to reduce apoptosis and increase differentiation into mature red blood cells. Although healthy individuals respond to anemia by increasing EPO levels up to 10,000-fold, patients with CKD have chronic inflammatory damage to renal EPO-producing cells and are often unable to generate sufficient levels of EPO to maintain adequate erythrocytosis. These individuals are often treated with synthetic forms of EPO (eg, epoetin, darbepoetin) to supplement intrinsic EPO production and stimulate erythropoiesis. Because synthetic forms of EPO can rapidly deplete iron stores (due to increased red blood cell production), patients should be tested for iron deficiency prior to treatment with these agents.

A 34-year-old woman comes to the office after her sister was diagnosed with breast cancer. She has no nipple discharge, breast lumps, or discomfort. Breast examination is normal. Her sister was found to have a multiple base pair insertion affecting exon 11 of the BRCA1 gene that leads to a frameshift mutation. A screening test to evaluate for a similar insertion mutation in the patient's BRCA1 gene is performed. The test uses polymerase chain reaction (PCR) to amplify the target exon and gel electrophoresis to assess the size of the exon compared to the wild-type allele. Which of the following must be known in order to perform the amplification part of this analysis? A.Restriction enzyme susceptibility sites within the target exon B.The amino acid sequence of the abnormal BRCA1 protein C.The cDNA nucleotide sequence D.The complete nucleotide sequence of the target exon E.The nucleotide sequence of the regions flanking the target exon

Polymerase chain reaction (PCR) is used to amplify small fragments of DNA (eg, genes, exons, noncoding regions) by repeated replication. Several elements are required for PCR. The first is a source DNA template that includes the target region to be amplified (eg, BRCA1 exon 11) as well as flanking sequences adjacent to the target region. The oligonucleotide sequence of these flanking regions must be known in order to make the primers necessary to start PCR. However, the exact nucleotide sequence of the target region does not need to be known (Choice D). A thermostable DNA polymerase (not denatured at high temperatures) is then used to replicate the DNA template from a pool of supplied deoxynucleotide triphosphates using the following 3 steps: Denaturing: Thermal separation of the DNA template is accomplished by exposing the sample to high temperatures Annealing: Primers combine with the denatured, single-stranded flanking ends of the target region when the temperature is lowered Elongation: DNA polymerase forms new daughter DNA strands in the 5' to 3' direction, starting from the 3' end of each primer These steps are repeated to obtain millions of copies of the target DNA segment in a short period of time (exponential amplification).

A 28-year-old man comes to the office at his wife's insistence. He reports severe insomnia but otherwise feels physically healthy. His wife is concerned that the patient is having a difficult time since returning from military duty. She says, "He used to be so upbeat and easygoing. Now his moods fluctuate between really tense and on edge to detached and numb." The patient cannot fall asleep at night because, as soon as he closes his eyes, he sees the horrific scene of his friend being blown up after stepping on a landmine. During the day, he is hypersensitive to loud sounds such as firecrackers or a car backfiring. The patient says, "Sometimes I feel as if I'm back in combat and have to duck and take cover." Vital signs are within normal limits, and physical examination shows no abnormalities. On mental status examination, the patient has an anxious mood and blunted affect. Which of the following is the most appropriate pharmacotherapy? A.Antipsychotic B.Benzodiazepine C.Monoamine oxidase inhibitor D.Mood stabilizer E.Non-benzodiazepine hypnotic F.Selective serotonin reuptake inhibitor G.Tricyclic antidepressant

SSRI Go watch sketchy if you got it wrong again

A 53-year-old man comes to the office with a dull ache in his right shoulder. The pain is worse with movement and often interferes with his sleep. The patient cannot recall any traumatic events prior to the start of the pain but did experience increased discomfort after helping his daughter move into her college dorm room a week ago. His past medical history includes dyslipidemia and hypertension, for which he takes the appropriate medications. Physical examination reveals localized tenderness just below the acromion. The physician asks the patient to abduct his arms 90 degrees to the side and flex them 30 degrees forward with his thumbs pointing to the floor. She then applies downward force to his arms. This maneuver elicits pain in the patient's right shoulder and reveals right-sided weakness as compared to the left side. A tendon of which of the following muscles is most likely injured in this patient? A) deltoid B) biceps brachii C) brachioradialis D) supraspinatus E) serratus anterior

The rotator cuff consists of the tendons of the supraspinatus, infraspinatus, teres minor, and subscapularis (SITS) and contributes to the stability and motion of the glenohumeral joint. During abduction of the humerus, the supraspinatus initiates movement through the first 10-15 degrees; subsequently, the deltoid provides the primary abductive force while the supraspinatus provides stability to the joint. Of all the rotator cuff structures, the supraspinatus tendon is most commonly affected in rotator cuff syndrome. This tendon is vulnerable to chronic repeated trauma from impingement between the head of the humerus and the acromion during abduction. Inflammation and fibrosis can worsen the problem by increasing friction between the head of the humerus and the acromion, as well as causing inflammation of the subacromial bursa. On examination, the action of the supraspinatus can be isolated with the "empty-can" supraspinatus test: abduction of the humerus in parallel to the axis of the scapula (30 degrees forward flexion) while in full internal rotation (thumbs pointed to the floor).

A 60-year-old man comes to the office due to difficulties in tasting food. He says, "All food now tastes bland," and he is unable to enjoy different flavors during meals. Two months ago, the patient had a traumatic brain injury after a head-on motor vehicle collision. He was in a coma for several days and, after regaining consciousness, began having severe headaches and impaired taste. The headaches have improved, but he continues to have difficulty tasting food. The patient has no visual concerns, slurred speech, trouble swallowing, vertigo, or extremity weakness or numbness. He is a truck driver and has a history of heavy tobacco use for many years. Vital signs are within normal limits. Physical examination shows no focal motor deficits. Which of the following is the most likely cause of this patient's current symptoms? A.Anterior temporal lobe contusion B.Avulsion of olfactory nerve rootlets C.Ischemic stroke of the dorsolateral medulla D.Lingual nerve injury E.Sphenoid bone fracture

This patient has a decreased perception of flavors after traumatic brain injury. The perception of flavor is dependent on both smell and taste, with smell being more important. Patients with anosmia (loss of smell) often describe difficulties with taste, even if taste sensation is intact. The perception of smell is mediated by odorant molecules that bind to olfactory chemoreceptor cells in the nasal mucosa. Axons of the chemoreceptor cells make up the olfactory nerve (CN I) rootlets, which project through the cribriform plate of the ethmoid bone to synapse on the glomeruli of the olfactory bulb. The olfactory bulb then projects second-order axons to the primary olfactory cortex in the medial temporal lobe via the olfactory tract. Head trauma can cause anosmia when acceleration-deceleration forces lead to avulsion of the olfactory nerve rootlets as they transverse the cribriform plate.

A 34-year-old man comes to the office due to a painless penile ulcer, which he first noticed 3 days ago. He had unprotected sexual intercourse with a new partner a few weeks ago. The patient has no significant medical history and takes no medications. Temperature is 37.1 C (98.8 F). Examination reveals a 2-cm nontender ulcer close to the glans penis with a raised, indurated margin and a clean base. There are no surrounding lesions or vesicles. There are several bilateral enlarged inguinal lymph nodes, which are firm, nontender, and rubbery. Physical examination is otherwise unremarkable. Rapid plasma reagin and HIV testing are negative. Infection with which of the following is the most likely cause of this patient's symptoms? A.Chlamydia trachomatis (L serovars) B.Haemophilus ducreyi C.Herpes simplex virus D.Neisseria gonorrhoeae E.Treponema pallidum

This patient developed a genital ulcer a few weeks after unprotected intercourse, raising suspicion for a sexually transmitted infection. A wide range of pathogens can cause genital ulcers, but a solitary, painless ulcer with heaped-up borders and a clean base usually indicates primary syphilis (syphilitic chancre). Syphilis is caused by Treponema pallidum, a gram-negative spirochete that cannot be cultured and cannot be detected on Gram stain due to its small size. The diagnosis of syphilis relies primarily on nontreponemal (eg, rapid plasma reagin, VDRL) and treponemal (eg, fluorescent treponemal antibody absorption) serologic testing. However, because humoral antibody response often takes 4 weeks to develop, false-negative serologic testing is common early in the course of disease. Nontreponemal tests are particularly susceptible to initial false-negative results (as in this patient); treponemal tests are more sensitive in early infection and are often used preferentially in this setting. Although not widely available, the diagnostic gold standard for syphilis is to directly visualize the organism in tissue scrapings from a cutaneous lesion (eg, chancre). This can involve fluorescent antibody testing or dark-field microscopy, which will show motile, helical spirochetes.

A 33-year-old man comes to the office due to fatigue and decreased exercise tolerance over the past 6 months. The patient was diagnosed with infective endocarditis in the setting of intravenous drug use a year ago and completed a course of antibiotic therapy. He has not used drugs since then. The patient has no other medical problems and takes no medications. A cardiac murmur is heard on auscultation. Both carotid arteries demonstrate a rapid rise and fall of the arterial pulse. Partial compression of the femoral arteries by the stethoscope elicits a systolic-diastolic bruit. Which of the following sets of pressure findings is most likely to be observed during cardiac catheterization of this patient? Question Id: 14976

This patient has aortic regurg so the pulse pressure is going to be wide. This patient's presentation is consistent with aortic regurgitation (AR), which likely developed as a complication of infective endocarditis. AR is characterized by a rapid loss of aortic pressure during diastole due to backflow of blood into the left ventricle. The left ventricle becomes dilated with an increase in end-diastolic volume and undergoes eccentric hypertrophy that allows for a compensatory increase in stroke volume and cardiac output. The result of these effects is a characteristic high-amplitude, rapid rise-rapid fall pulsation with each ventricular contraction that manifests as widened pulse pressure, rapid distension and collapse of the carotid arteries (Corrigan sign), and a "to-and-fro" bruit appreciated over the femoral arteries (Duroziez sign). The hemodynamic changes in AR are evident on cardiac catheterization and include reduced aortic diastolic pressure and elevated left ventricular (LV) diastolic pressure due to backflow of blood during diastole. In addition, aortic and LV systolic pressures are increased due to high stroke volume and a compensatory increase in cardiac contractility.

A 65-year-old man is being evaluated in the hospital. The patient was admitted 5 days ago for increasing lower extremity edema and dyspnea. Medical history is significant for obesity hypoventilation syndrome, pulmonary hypertension, and chronic lower extremity edema. Current temperature is 37.2 C (99 F), blood pressure is 110/70 mm Hg, pulse is 90/min, and respirations are 16/min. BMI is 50 kg/m2. Laboratory results are as follows: AdmissionToday (5th day) Hemoglobin 13.1 g/dL to 14.5 g/dL Blood glucose 98 mg/dL to 90 mg/dL Blood urea nitrogen 24 mg/dL to 64 mg/dL Serum creatinine 1.2 mg/dL to 2.1 mg/dL Urinalysis negative for protein, red blood cells, white blood cells, and casts Urine sodium now 5 days later is 10 mEq/L Which of the following is the most likely cause of the laboratory abnormalities in this patient? A) osmotic diuresis B) diuretic therapy C) interstial nephritis D) uretreal obstruction

This patient has volume overload due to right-sided heart failure from pulmonary hypertension; he was most likely treated with diuretics to improve his volume status. However, laboratory results on day 5 demonstrate an abrupt decrease in renal function with low urine sodium and an elevated BUN/creatinine ratio (prerenal azotemia). In association with a normal urinalysis, this presentation suggests volume depletion from the excessive use of diuretics. Excessive diuresis can cause renal hypoperfusion and activation of the renin-angiotensin-aldosterone system (RAAS), which increases solute and water reabsorption by the kidney in an attempt to restore intravascular volume. As a result, laboratory studies will show low urine sodium (<20 mEq/L) and low fractional excretion of sodium (FENa <1%). Urea reabsorption is also increased in the collecting ducts, resulting in an increased BUN/creatinine ratio (>20:1). Elevated urine osmolality (>450 mOsm/kg) and urine specific gravity (>1.015) reflect concentrated urine. Evidence of hemoconcentration (eg, increased hemoglobin, albumin, uric acid levels) is also common in volume depleted patients.

A 36-year-old man is rushed to the emergency department after sudden onset of shortness of breath and difficulty swallowing. He has visited the emergency department several times before due to food and skin allergies. The patient is unconscious on arrival. Examination shows excessive accessory respiratory muscle use and edematous swelling of the face, lips, and tongue. There is also scattered urticaria over the upper body. Attempts at intubation are unsuccessful due to massive soft tissue edema involving the pharynx. A decision is made to perform an emergency cricothyrotomy. The incisions made during this procedure will most likely pass through which of the following structures?

This patient is experiencing acute, life-threatening laryngeal edema as part of an anaphylactic reaction precipitated by exposure to an allergen. Laryngeal edema can initially present with throat tightness, difficulty swallowing, dyspnea, and hoarseness. However, the condition can quickly progress to compromise the airway and cause asphyxiation. Cricothyrotomy is indicated when an emergency airway is required and orotracheal or nasotracheal intubation is either unsuccessful or contraindicated (eg, massive hemorrhage, vomiting, facial trauma, airway obstruction). The procedure establishes an airway through the placement of a tube between the cricoid and thyroid cartilages and requires incision through the following structures: 1)Skin 2) Superficial cervical fascia (including subcutaneous fat and platysma muscle) 3) Investing and pretracheal layers of the deep cervical fascia 4) Cricothyroid membrane

A 55-year-old man comes to the emergency department due to an hour of intense substernal chest burning accompanied by sweating. He has a history of obesity and diet-controlled type 2 diabetes mellitus. On initial evaluation, blood pressure is 110/70 mm Hg and pulse is 60/min and regular. Oxygen saturation is 98% on room air. The patient is comfortable lying flat and no heart murmurs are heard. Lungs are clear on auscultation. Immediate ECG shows normal sinus rhythm with a 2-mm ST-segment elevation in leads II, III, and aVF. Aspirin and morphine are administered. As the patient is about to be transported to the catheterization lab for urgent percutaneous coronary intervention, he develops dizziness and the telemetry monitor shows sinus bradycardia at 40/min. The patient appears pale and diaphoretic and peripheral pulses are faint. Blood pressure is 70/40 mm Hg. Which of the following is the best immediate step for managing this patient's condition?

This patient with ST elevation in leads II, III, and aVF has an acute inferior wall myocardial infarction (MI), which most commonly occurs due to occlusion of the right coronary artery (RCA). The RCA usually supplies blood to the sinoatrial node, the atrioventricular node, and most of the bundle of His; therefore, conduction impairment (eg, sinus bradycardia, atrioventricular block) is common in patients with inferior wall MI. Sinus bradycardia is the most common arrhythmia in those with inferior wall MI (nearly 50% of patients); it occurs due to both nodal ischemia and enhanced vagal tone triggered by infarction of myocardial tissue. The bradycardia is usually transient and resolves with restoration of coronary blood flow; however, patients with hypotension or symptoms of bradycardia (eg, lightheadedness, syncope) should be managed with intravenous atropine. The anticholinergic effects of atropine counteract the enhanced vagal tone to increase heart rate and improve symptoms.

A 43-year-old man undergoes induction chemotherapy for acute myeloid leukemia. Three weeks later, he is hospitalized with fever, cough, and generalized weakness. On physical examination, his temperature is 38.1 C (100.7 F), blood pressure is 102/68 mm Hg, and pulse is 125/min. The patient's pulse oximetry shows 95% on room air. Crackles are heard over the right upper lung field. His hemoglobin level is 9.5 g/dL, and leukocyte count is 900/mm3 with an absolute neutrophil count of 100/mm3. Chest x-ray reveals a dense infiltrate involving the right upper lung lobe. Broad-spectrum antibiotic therapy is instituted. Five days later, the patient is still febrile. Bronchoscopy with biopsy is performed and the tissue samples from the bronchoscopy grow mold. Which of the following is the most likely isolate? A) aspergillus B) Pneumocystis jiroveci C) cryptococcus D) candida

This patient with leukemia recently underwent chemotherapy (with likely myelosuppressive effects) and now has a fungal respiratory infection with febrile neutropenia (FN), typically defined as fever with an absolute neutrophil count <500/mm3. Patients with FN are at increased risk for severe infection. Broad-spectrum antibiotic therapy is indicated. Gram-positive organisms are now the most commonly identified pathogens in FN. However, patients with profound and prolonged neutropenia (several days) are at especially high risk for viral and fungal infections. Invasive pulmonary aspergillosis presents with some combination of fever, chest pain, cough, dyspnea, and hemoptysis. Chest imaging can show consolidation, nodules, or cavitary lesions. On light microscopy, Aspergillus is seen as septate narrow hyphae that branch at a 45° angle. Antifungal therapy is needed.

A 55-year-old man comes to the office due to several months of progressive fatigue. He does not have chest pain or shortness of breath. Medical history is significant for hypertension and obesity. BMI is 45 kg/m2. Examination shows mild mucosal pallor. Laboratory results are as follows: Complete blood count Hemoglobin8.6 g/dL Mean corpuscular volume84 µm3 Reticulocytes0.2% Platelets200,000/mm3 Leukocytes7,500/mm3Serum chemistry Blood urea nitrogen14 mg/dL Creatinine0.8 mg/dL Which of the following is the most likely cause of this patient's anemia? A.Anemia of chronic inflammation B.Extravascular hemolysis C.Microangiopathic hemolytic anemia D.Occult blood loss E.Vitamin B12 deficiency

This patient with morbid obesity has normocytic anemia with a low reticulocyte count, raising strong suspicion for anemia of chronic disease (ACD). Because ACD is triggered by long-standing elevations in serum inflammatory cytokines, it is classically associated with underlying rheumatologic disease, persistent infection, or malignancy. However, ACD is also commonly caused by less obvious sources of inflammation such as type 2 diabetes mellitus, congestive heart failure, severe traumatic injury, and morbid obesity. The major inflammatory cytokine that drives ACD is hepcidin, a small peptide released from the liver in response to inflammation or bacterial lipopolysaccharide. Hepcidin binds to and inhibits iron channels (ferroportin) on the surface of enterocytes and reticuloendothelial macrophages, leading to decreased iron absorption in the gut and decreased iron release by the reticuloendothelial system (which supplies ~95% of daily iron for erythropoiesis). Because heme synthesis is impaired without sufficient iron, patients with ACD typically develop a normocytic (or slightly microcytic) anemia with a minimal reticulocyte response.

A longitudinal study is conducted to assess changes in renal function over time in patients with recently diagnosed type 2 diabetes mellitus. An initial set of laboratory tests is obtained in newly enrolled patients to establish baseline renal function parameters. The following measurements are taken from a 42-year-old male volunteer. Serum Creatinine 110.0 mg/dL urine 1 .1 mg/dL Glucose 80.0 mg/dL Potassium 50.0 mEq/L urine 4.0 mEq/L Uric acid 15.0 mg/dL urine 3.0 mg/dL Para-aminohippuric acid 100 mg/mL urine 0.2 mg/mL Which of the following is the best estimate of the filtration fraction in this patient assuming a urine flow of 1.0 mL/min? A.10% B.20% C.30% D.40% E.50%

b Question Id: 1555

A 34-year-old man comes to the emergency department due to a facial injury. He reports getting hit on the face during a fistfight at a bar. Examination shows dark blue periorbital ecchymosis on the right side. Ophthalmic and neurologic examinations are otherwise normal. After appropriate evaluation, the patient is discharged home. Several days later, the bruise becomes greenish in color. This change in color is best explained by the activity of which of the following enzymes? A) bilirubin glucornytransferase B) heme oxygenase C) urophorphyrinogen deaminase D) ferrochetalase

b This patient has a resolving hematoma after a traumatic injury. Following the injury, hemoglobin-containing erythrocytes escape into the periorbital tissues, giving the bruise its initial purple or bluish color. Erythrocyte destruction causes the release of iron-containing heme molecules. Heme oxygenase (contained in macrophages, among other cells) degrades heme into biliverdin, carbon monoxide, and ferrous iron while consuming oxygen and electrons provided by NADH and NADPH-cytochrome P450 reductase. Biliverdin is green in color and is further reduced (by the enzyme biliverdin reductase) to the yellow pigment bilirubin, which is then transported to the liver bound to albumin.

A 10-year-old boy is brought to the emergency department for new swelling in his right leg. He has a history of lens dislocation and intellectual disability. Physical examination demonstrates moderate, pitting edema from his right calf to his right thigh and a normal left lower extremity. In addition, the patient has a caved-in appearing chest wall. He has no family members with similar conditions. Ultrasound reveals a deep venous thrombosis in his right femoral vein. Further genetic testing reveals a single missense mutation in the gene coding for cystathionine beta-synthase enzyme. Which of the following is the most likely explanation for this patient's genetic defect affecting multiple tissues? A) locus heterogenisity B) pleiotrophy

b This patient presenting with skeletal abnormalities, lens dislocation, intellectual deficits, vascular thromboses, and a genetic defect in the cystathionine beta-synthase enzyme likely has homocystinuria. The occurrence of multiple, seemingly unrelated phenotypic manifestations, often in different organ systems, as a result of a single genetic defect is termed pleiotropy. Most syndromic genetic illnesses including homocystinuria exhibit pleiotropy. Locus heterogeneity refers to the ability of one disease or trait to be caused by mutations in multiple different genes. An example is familial hypercholesteremia, which can be caused by different mutations affecting cholesterol metabolism genes (eg, LDL receptor, apo B-100).

A 43-year-old Caucasian female complaining of reduced energy and fatigue is found to have hypochromic, microcytic anemia. Her past medical history is significant for stable angina treated with metoprolol and aspirin. Iron supplementation is prescribed. Several weeks later, a peripheral blood smear demonstrates numerous enlarged red blood cells that appear blue on Wright-Giemsa stain. The bluish color of these red blood cells is best explained by the presence of which of the following? A) hemoglobin percipitates B)Ribosmol rRNA C)Nuclear membrane

b This patient suffers from hypochromic, microcytic anemia most likely caused by iron deficiency. Iron-deficient individuals on replacement therapy should experience hemoglobin level increases of approximately 2 g/dL per week for the first three weeks. This increase in hemoglobin results from enhanced erythropoiesis and the accelerated release of both mature red blood cells (RBCs) and reticulocytes into the bloodstream. The reticulocyte is an immature RBC that is slightly larger and bluer than a mature RBC. It lacks a cell nucleus but retains a basophilic, reticular (mesh-like) network of residual ribosomal RNA. The ribosomal RNA appears blue microscopically after the application of the Wright-Giemsa stain. After spending a day or so in the bloodstream, the reticulocytes are transformed into mature red blood cells that have a lifespan of approximately 120 days.

A 45-year-old man comes to clinic due to frequent episodes of palpitations accompanied by dizziness, fatigue, and shortness of breath. Prolonged ECG monitoring identifies episodes of atrial fibrillation associated with a rapid ventricular response rate. A radiofrequency ablation procedure is planned. The access site is the right femoral vein. The ablation catheter is advanced to the left atrium where radiofrequency energy is used to eliminate an ectopic focus of abnormal electrical activity. During the procedure, the catheter most likely passes through which of the following structures? A) Aortic valve B) Interatrial septum C) Interventricular septum D) Pulmonic valve E) Tricuspid valve

b A venous catheter traveling from the femoral vein to the heart passes through the iliac vein and inferior vena cava to reach the right atrium. Once in the right atrium, structures within the right side of the heart and the pulmonary arteries are readily accessible. However, because the pulmonary capillaries are far too small to pass through, the left side of the heart must be accessed by traversing the interatrial septum. The interatrial septum is traversed at the site of the foramen ovale, which in adults is typically covered by a thin membrane of fibrous tissue that can be easily punctured.

A 63-year-old woman comes to the emergency department with fever and a cough. A new medical student rotating in the emergency department is sent to interview and examine the patient. The patient reports 3 months of fevers, night sweats, and a productive cough, and is subsequently placed in an isolation room. A chest x-ray demonstrates a right apical lung lesion. The medical student has never been exposed to Mycobacterium tuberculosis and reports that he was in close contact with the patient, who coughed frequently during their interaction. Which of the following would most likely happen during the first week after exposure? A. Epithelioid transformation of macrophages B. Interferon secretion by activated T lymphocytes C. Intracellular bacterial proliferation D. Mounting response by B lymphocytes E. Scattered areas of caseous necrosis

c Mycobacterium tuberculosis is a facultative intracellular, acid-fast bacillus that is transmitted via aerosolized droplet nuclei. The pathogen is inhaled into the lower lobe of the lung and replicates within the alveoli. Alveolar macrophages phagocytose the organism but are initially unable to eliminate it due to microbial virulence factors (eg, cord factor) that prevent phagolysosome fusion and acidification. This allows M tuberculosis to replicate in an unchecked fashion within the phagosome of alveolar macrophages for the first few weeks of the infection. After a few weeks, antigen-presenting cells in the draining lymphatic system display mycobacterial antigens and release interleukin-12, which stimulates naïve CD4 lymphocytes to differentiate into T helper type 1 (TH1) cells. Th1 cells release interferon-gamma (Choice B), which activates macrophages and causes them to: Form fully acidified phagolysosomes that are capable of destroying intracellular mycobacteria Differentiate into epithelioid cells and multinucleated Langhans giant cells that surround extracellular mycobacteria within granulomas (Choice A). These cells release digestive enzymes and generate reactive oxygen species, which helps limit mycobacterial proliferation but also cause caseating necrosis and (in some patients) cavitary lung lesions (Choice E).

A 28-year-old man is hospitalized due to the acute onset of severe depression and suicidal ideation for several days. He was recently thrown out of his mother's home after stealing her money to support his drug habit. The patient reports fatigue and vivid, disturbing dreams. The nursing staff note that he is irritable, withdrawn, hypersomnolent, and hyperphagic. The patient has an extensive history of substance abuse and has been hospitalized previously for alcohol detoxification. Blood pressure is 110/80 mm Hg and pulse is 64/min. Physical examination shows old injuries from a motorcycle accident but no other abnormalities. On mental status examination, he appears lethargic and dysphoric. This patient's current symptoms are most likely the result of which of the following conditions?

cocaine withdrawal

A 42-year-old woman, gravida 4, para 4, comes to the clinic due to heavy and painful menstrual bleeding over the past 3 months. Her last menstrual period was 4 weeks ago. Menarche was at age 10, and menstrual periods last for 3-5 days and occur every 30 days. She is sexually active with her husband and does not have pain with intercourse. The patient had a bilateral tubal ligation 3 years ago after the birth of her last child. She takes no medications and has no allergies. BMI is 24 kg/m2. Vital signs are normal. On bimanual examination, the uterus is uniformly enlarged. Urine β-hCG is negative. Biopsy shows secretory endometrium. Which of the following is the most likely cause of this patient's symptoms? A) Benign myometrial smooth muscle cell proliferation B) Blastocyst implantation in the fallopian tube C) Deficiency of von Willebrand factor D) Endometrial tissue in the myometrium E) Greater increase in endometrial gland proliferation compared to stroma F) Hyperplastic growth of tissue from endometrial surface

d Adenomyosis is the presence of endometrial glandular tissue within the myometrium. The condition is relatively common in middle-aged parous females. Symptoms include heavy menstrual bleeding, which is due to an increased endometrial surface, and dysmenorrhea, which results from endometrial tissue growth in the confined myometrial space. Physical examination classically shows a uniformly enlarged uterus (as seen in this patient). Uterine enlargement results from hormonal stimulation of endometrial glandular tissue in the myometrium. An endometrial biopsy is a sample of the tissue in the endometrial lining, and therefore, this patient's biopsy result of secretory endometrium is a normal finding for a woman in day 21 of her menstrual cycle. Adenomyosis can only be diagnosed definitively by microscopic examination of a hysterectomy specimen.

A study aims to investigate the effectiveness of the topical application of tetracycline ointment in the prevention of surgical-site infection associated with resection of advanced oral cavity cancer. Fifty patients who underwent locoregional resection were assigned by chance to receive standard care alone or tetracycline-ointment in addition to standard care. The patients were then followed for 1 month and the number and severity of surgical-site infections were recorded for each group. Which of the following best describes the study design? A.Case-control study B.Crossover design C.Prospective cohort study D.Randomized control trial E.Retrospective cohort study

d Research studies are classified as observational studies (eg, cross-sectional, case control, cohort) or experimental studies (eg, randomized control trials, factorial, crossover), depending on how much control a researcher has over the independent variables (eg, risk factors/exposures, interventions) in a study. The main differences between these 2 types of study designs are as follows: In observational designs, the researcher observes the effect of naturally occurring risk factors/exposures on outcomes of interest. In experimental designs, the researcher randomly assigns interventions to potential participants to assess the effect of the controlled interventions. A randomized control trial is one of the simplest subtypes of experimental designs; it is an experiment in which participants are randomly allocated to ≥2 groups to assess the effect of specific interventions (eg, treatments). In this example, 50 patients who underwent surgical resection of advanced oral cavity cancer were assigned by chance into either a tetracycline-ointment treatment group or a standard-of-care control group. The effectiveness of the topical application of tetracycline ointment for prevention of surgical-site infection was then assessed by recording the incidence and severity of infections in each group.

A 43-year-old woman comes to the office due to episodic confusion, blurred vision, tremors, and sweating. The episodes have increased in intensity in the past few months and seem to occur any time during the day. Her symptoms are relieved by drinking orange juice. During a recent episode, the patient's blood glucose level was 32 mg/dL as measured by her husband's home glucose meter. Medical history includes major depression and irritable bowel syndrome. Laboratory studies performed during the next episode of symptoms show the following results: Glucose 35 mg/dL Insulin 18 µU/mL(normal: <3 µU/mL) C-peptide 4.01 ng/mL(normal: <0.6ng/mL) Which of the following is the most likely cause of this patient's symptoms? A.Alpha cell tumor B.Insulin administration C.Somatostatinoma D.Sulfonylurea use E.VIP-secreting tumor

d This patient has symptomatic hypoglycemia, confirmed by the Whipple triad: Symptoms consistent with hypoglycemia (eg, tremor, diaphoresis, confusion) Low blood glucose level Relief of hypoglycemic symptoms when the blood glucose level is corrected Insulin is initially synthesized as preproinsulin, after which the signal peptide is removed, yielding proinsulin. In the secretory granules, proinsulin is cleaved into insulin and C-peptide, which are then released together in equimolar amounts. Normally, as blood glucose falls, endogenous insulin secretion is suppressed, and serum levels of both insulin and C-peptide drop. Hypoglycemia due to excessive endogenous insulin production is associated with elevated levels of C-peptide. This can occur due to autonomous insulin-producing tumors (insulinomas) or intake of insulin secretagogues (eg, sulfonylureas). Self-induced hypoglycemia via oral hypoglycemics is often a manifestation of a factitious disorder and is typically seen in health care workers and family members of patients with diabetes using these medications (which is likely in this patient whose husband has home glucose-testing supplies). Sulfonylurea abuse can be confirmed by testing the urine or blood for specific hypoglycemic agents.

An 8-month-old girl is brought to the office for evaluation of irritability and regression of motor skills. Her birth was unremarkable and she appeared to develop normally, but she can no longer sit or roll over. Her parents have also noticed that she startles easily with loud noises. Head circumference measurement is consistent with macrocephaly. Bilateral funduscopic evaluation shows a bright red fovea centralis that is surrounded by a contrasting white macula. Peripheral vision is decreased. Abdominal examination is normal. Accumulation of which of the following metabolites is most likely present in this patient's tissues? A.Galactocerebroside B.Globotriaosylceramide C.Glucocerebroside D.Glycogen E.GM2 ganglioside F.Heparan sulfate G.Sphingomyelin

e taysachs disease

Electric stimulation of the musculocutaneous nerve causes a rapid twitch of the biceps muscle. Electron microscopy shows numerous vesicles within the musculocutaneous nerve terminals. In order for these vesicles to release their contents into the synaptic cleft, which of the following substances is required? A.Sodium B.Calcium C.cAMP D.IP3 E.DAG F.cGMP

go check out the first aid muscle physio picture B The synaptic vesicles in motor neuron presynaptic terminals contain acetylcholine (ACh). When an action potential depolarizes the presynaptic terminal membrane, voltage-gated calcium channels open, causing an influx of extracellular calcium into the synaptic terminal. This calcium current stimulates fusion of the presynaptic vesicles with the plasma membrane and exocytosis of ACh. The nicotinic ACh receptors on the postsynaptic skeletal muscle end plate are ligand-gated sodium and potassium ion channels. The increased sodium and potassium conductance that occurs with ACh binding generates a motor end plate potential that signals for depolarization of the muscle cell membrane. ACh in the synaptic cleft is degraded by acetylcholinesterase.

A 58-year-old man comes to the emergency department due to vision disturbances. Over the past several weeks, the patient has had intermittent episodes in which he sees everything tinted with blue. His vision returned to normal within a few hours each time. When he woke up today, vision in the left eye was blurry. He has had no eye pain, conjunctival redness, or headache. The patient has tingling and numbness in the feet due to diabetic neuropathy, which is unchanged, and reports no other focal weakness or sensory loss. Medical history is notable for hypertension, type 2 diabetes mellitus, hyperlipidemia, and erectile dysfunction. Vital signs are within normal limits. Physical examination shows an afferent pupillary defect, decreased visual acuity, and optic disc edema in the left eye. Discontinuing which of the following medications may improve this patient's vision? MOA?

phosphodiesterase inhibitor slidenafil Normal male erectile function involves interplay between psychological and physical stimulation and the vascular, neurologic, and hormonal systems. On a vascular level, penile erection is induced by increased blood flow to the corpora cavernosa and corpus spongiosum accompanied by decreased outflow. Endothelial factors required for erection include formation of nitric oxide, which induces formation of cyclic GMP (cGMP), which in turn mediates vascular smooth muscle relaxation. cGMP is inactivated by cGMP phosphodiesterase (PDE) (ie, PDE5), which cleaves cGMP and terminates penile erection. PDE5 inhibitors (eg, sildenafil, tadalafil) are widely used, first-line agents for treatment of erectile dysfunction (ED). PDE5 inhibitors increase activity of cGMP in the penile corpora, leading to greater and more prolonged tumescence in men with ED. However, PDE5 inhibitors can also inhibit PDE6 in the retina, which is involved in color vision. Patients experience this effect as a transient bluish discoloration to vision. Less common ocular effects of PDE5 inhibitors include nonarteritic anterior ischemic optic neuropathy, which presents with sudden monocular vision loss associated with an afferent pupillary defect, decreased visual acuity, and optic disc edema.

A 35-year-old man comes to the office due to a lack of sexual interest. He says, "I started seeing this woman 9 months ago and am really attracted to her. We get along well and things are great, but I just don't want to have sex with her anymore." On further questioning, the patient reluctantly admits that whenever they have intercourse he has an orgasm in less than a minute and finds this very embarrassing. He does not have the same problem when he masturbates. He says, "I'm worried that my girlfriend will leave me, and it's really affecting my self-esteem." The patient has no other concerns. His other medical conditions include type 1 diabetes mellitus, chronic insomnia, and a history of major depression. His medications include insulin glargine, short-acting insulin, and trazodone. He does not use illicit substances. Vital signs are within normal limits. Laboratory results are significant for hemoglobin A1c of 7% and fasting glucose of 130 mg/dL. Which of the following is the most likely diagnosis?

premature ejeculation This patient has features of premature ejaculation, characterized by unwanted episodes of early ejaculation accompanied by a sense of lack of control. Although concern over the time to ejaculation is common, only an estimated 4% of men will meet diagnostic criteria (ejaculation within one minute of penetration, occurring most of the time for at least 6 months). The diagnostic criteria for premature ejaculation are based on ejaculation during partnered sexual activity, therefore a normal time to ejaculation during masturbation does not negate this diagnosis.

A 30-year-old woman is evaluated for a 3-month history of progressive fatigue, decreased appetite, and 10-lb (4.5-kg) weight loss. The patient has type 1 diabetes mellitus and has noticed decreased insulin requirements over this time. She has no other medical conditions and does not use tobacco, alcohol, or illicit drugs. Physical examination shows a generalized increase in pigmentation of the skin, especially involving the palmar creases. Measurement of serum cortisol before and after administration of exogenous adrenocorticotropic hormone (ACTH) shows no difference in the levels. Which of the following changes in serum laboratory values is most likely present in this patient? what will happen to sodium, k, cl and bicarb

sodium decrease, potassium increase, cl increase, and bicarb decreased This patient's fatigue and weight loss are likely due to primary adrenal insufficiency (Addison disease). Type 1 diabetes mellitus is caused by autoimmune destruction of pancreatic beta cells and is associated with an increased risk of other autoimmune endocrinopathies. Autoimmune adrenalitis is the most common cause of primary adrenal insufficiency and results from autoantibodies against all 3 zones of the adrenal cortex. Because the main defect is in the adrenal gland, exogenous ACTH administration does not increase cortisol levels. Decreased negative feedback of cortisol on the pituitary gland increases release of ACTH and melanocyte-stimulating hormone, resulting in hyperpigmentation. Patients with diabetes often have reduced insulin requirements due to decreased appetite, weight loss, and increased insulin sensitivity (glucocorticoids decrease insulin sensitivity). Hypoaldosteronism in primary adrenal insufficiency leads to decreased Na+ reabsorption and decreased urinary excretion of K+ and H+. Decreased H+ excretion causes a nonanion gap metabolic acidosis with low plasma HCO3-. This leads to compensatory Cl- retention to maintain electrical neutrality of the extracellular fluid. The hypotension induced by lack of aldosterone stimulates vasopressin release, increasing free water reabsorption and further exacerbating hyponatremia. As a result, patients with primary adrenal insufficiency present with hyponatremia, hyperkalemia, hyperchloremia, and nonanion gap metabolic acidosis.

A 12-year-old boy is evaluated due to fever, fatigue, and easy bruising. Physical examination shows pallor, generalized lymphadenopathy, and bone tenderness. Laboratory evaluation reveals anemia, thrombocytopenia, and leukocytosis with atypical lymphoblasts. Bone marrow biopsy is consistent with acute lymphoblastic leukemia. The patient is treated with combination chemotherapy, and 6-mercaptopurine is administered for maintenance of remission. Which of the following enzymes inactivates this drug? 2 enzymes?

xanthine oxidase and thiopurine methytransferase 6-mercaptopurine (6-MP) can be administered directly or in its prodrug form (ie, azathioprine). Following ingestion, 6-MP is converted by hypoxanthine-guanine phosphoribosyltransferase (HGPRT) into 6-thioguanine (6-TG) active metabolites. The 6-TG metabolites are purine analogues that act as false nucleotides to disrupt DNA and RNA synthesis and inhibit the proliferation of hematopoietic cells, a useful effect in the treatment of autoimmune disease, organ transplant rejection, and some types of cancer (eg, leukemia). 6-MP is metabolized into inactive metabolites by 2 different enzymes, xanthine oxidase and thiopurine methyltransferase (TPMT), both of which are clinically important. Coadministration of 6-MP with a xanthine oxidase inhibitor (eg, allopurinol, febuxostat) slows the inactivation of 6-MP and shunts metabolism toward the production of 6-TG active metabolites; therefore, the dose of 6-MP must be significantly reduced. In addition, roughly 12% of the population has a genetic mutation resulting in low or absent activity of TPMT, creating a high risk of toxicity with the administration of 6-MP. Therefore, some clinicians recommend that patients undergo genetic testing for TPMT activity prior to initiating 6-MP (or azathioprine) therapy.

A group of psychiatrists has noticed an unusual number of older individuals with depression in the surrounding community. A decision is taken to conduct a cross-sectional study to estimate the prevalence of depression in this specific population. Estimates for the prevalence of depression (cases per 100 individuals) in the community are as follows: Age group Women Men 60-69= 9.1, men 8.4 70-79= 11.9 men 10.4 80-89= 13.5 men 12.4 90+ = 13.3 men 12.1 Total=11.9 men total =10.8 Suppose during an afternoon clinic, a psychiatrist sees a 65-year-old woman, a 72-year-old man, and an 84-year-old woman from this community. Assuming no relation between the 3 patients, what is the probability that none of them has depression?

0.705 find the probablity of not having the disease then multiply each The age- and gender-specific prevalence of depression in the community is a good estimator of the probability that a corresponding individual from community has the disease. Because each of the 3 patients is unrelated to the others, each of the diagnoses is an independent event (ie, a patient's diagnosis has no impact on another's). When events are independent of each other, the probability that all the events occur is calculated by multiplying together the probability of each individual event. In this case, to calculate the probability that none of the 3 patients has depression, multiply the probability of each complementary event (ie, the probability of not having depression): (1 − 0.091) × (1 − 0.104) × (1 − 0.135) = 0.909 × 0.896 × 0.865 = 0.705

A 64-year-old African American woman is brought to the clinic by her husband due to a fall. She was walking in her living room when she tripped over an electrical cord. The patient has had several other ground-level falls in the last 6 months but has had no significant injuries. She also gave up driving a year ago following a near-miss motor vehicle accident. Past medical history is notable for hypertension and type 2 diabetes mellitus, for which she takes appropriate medications. Mental status and neurologic examinations are normal. Ophthalmic examination reveals reduced vision in her peripheral visual fields bilaterally and an elevated intraocular pressure. After discussion of treatment options, the patient is initiated on timolol ophthalmic drops. Which of the following structures is the most likely target of this medication? A.Ciliary epithelium B.Ciliary muscle C.Lens D.Pupil sphincter E.Trabecular meshwork

A go check out sketchy Open-angle glaucoma is characterized by progressive loss of peripheral vision from elevated intraocular pressure. Timolol and other nonselective beta blockers work by diminishing the secretion of aqueous humor by the ciliary epithelium. Acetazolamide, a carbonic anhydrase inhibitor, also decreases aqueous humor secretion by the ciliary epithelium. Prostaglandin F2α (eg, latanoprost, travoprost) and cholinomimetics (eg, pilocarpine, carbachol) decrease intraocular pressure by increasing the outflow of aqueous humor.

A 20-year-old man comes to the office for evaluation of abnormal behavior. Over the past 4 months, the patient has had several episodes of unresponsiveness, during which he would stare blankly while smacking his lips. He spontaneously becomes responsive after about 2 minutes and has no recollection of the event. The patient says these episodes are often preceded by an uneasy sensation in his upper abdomen and the pungent smell of something burning. He has had no falls or jerky movements of the extremities. The patient has no history of recent fever, headaches, head trauma, nausea, vomiting, or focal neurologic deficit. He had seizures in early childhood precipitated by high fevers but has had none since then and reports no other medical conditions. Cranial nerve examination reveals no abnormalities. Motor strength, sensation, and deep tendon reflexes are normal throughout. Neuroimaging is most likely to reveal an abnormality in which of the following locations in the brain? A) Hippocampus B) parietal lobe C) occipital D) Premotor frontal cortex

A this is temporal lobe epilepsy- the patient is expriencing focal seizure Focal seizures originate in a single hemisphere (vs generalized seizures, which originate throughout the cerebral cortex) and are subcategorized as: Focal aware seizures (previously simple partial seizure): Patients are aware and interactive but have symptoms corresponding to the involved area of the brain (eg, face or limb twitching/jerking - frontal lobe motor cortex, auditory/olfactory hallucinations - temporal lobe, visual phenomena - occipital lobe). Focal impaired awareness seizures (previously complex partial seizures): Patients appear awake but do not interact appropriately (eg, blank stare) and frequently engage in repetitive, stereotyped automatisms (eg, lip smacking, hand wringing, repeating words). Focal seizures with impaired awareness are the typical manifestation of temporal lobe epilepsy, the most common type of recurrent seizure disorder. Seizures are often preceded by a distinctive aura (eg, uneasy epigastric sensation, unpleasant olfactory hallucinations) that localizes the origin of the epileptiform discharges to the mesial temporal lobe (eg, hippocampus, amygdala, parahippocampal gyrus). The disorder is usually due to underlying hippocampal sclerosis (ie, mesial temporal sclerosis), which is associated with childhood febrile seizures.

An 82-year-old man is brought to the emergency department after a syncopal episode. He has no chest pain or dyspnea but has severe constipation of recent onset. He was hospitalized 2 weeks ago for atrial fibrillation with rapid ventricular response and was discharged home with oral medications after appropriate management. His past medical history is also significant for hypertension and severe chronic obstructive pulmonary disease requiring oxygen therapy. His blood pressure is 105/60 mm Hg and pulse is 50/min. Examination reveals bilateral decreased breath sounds, no wheezing, and normal heart sounds. ECG shows new-onset second-degree atrioventricular block. Which of the following drugs is the most likely cause of his current condition? A.Amlodipine B.Diltiazem C.Indapamide D.Lidocaine E.Propranolol F.Terazosin G.Valsartan

B go watch pharm video on CCB This patient's severe constipation and new-onset second-degree atrioventricular (AV) block (causing syncope) in the setting of new medication use for atrial fibrillation (AF) is likely due to calcium channel blocker (CCB) therapy. Diltiazem and verapamil are nondihydropyridine CCBs that are frequently used for hypertension, angina pectoris, and supraventricular arrhythmias (atrial flutter, AF, paroxysmal supraventricular tachycardia). These drugs exert their primary action by blocking the L-type calcium channels, thereby decreasing phase 0 depolarization and conduction velocity in the sinoatrial and AV nodes. This leads to slowing of the sinus rate and conduction through the AV node, which can then cause bradycardia and varying degrees of AV block. The drugs also have a negative inotropic effect and are relatively contraindicated in patients with congestive heart failure due to left ventricular systolic dysfunction. Constipation is a major side effect of nondihydropyridine CCBs (verapamil > diltiazem).

A 62-year-old man is hospitalized with severe abdominal pain and diarrhea after a recent urinary tract infection. Six months ago, the patient had an episode of Clostridium difficile colitis after being treated for pneumonia. Medical history is also significant for diverticulitis and upper gastrointestinal tract bleeding. The patient is allergic to penicillin. An appropriate workup confirms C difficile colitis. The patient is placed on an oral macrocyclic antibiotic that inhibits the sigma subunit of RNA polymerase. Which of the following agents was most likely administered to this patient? A.Doxycycline B.Fidaxomicin C.Metronidazole D.Neomycin E.Vancomycin

B Treatment options for Clostridium difficile infection (CDI) typically include oral vancomycin or fidaxomicin. Fidaxomicin is a macrocyclic antibiotic (related to macrolides) that inhibits the sigma subunit of RNA polymerase, leading to protein synthesis impairment and cell death (bactericidal activity against C difficile). It is administered orally and has minimal systemic absorption, resulting in high fecal concentrations. It also has a narrow spectrum of activity with a lesser effect on normal colonic flora than vancomycin.

A 46-year-old man comes to the emergency department due to fever. He returned from a vacation in Southeast Asia a week ago and began having symptoms toward the end of his trip. The patient describes continuous fever that has risen slowly to reach a high plateau and is relieved only by antipyretics. He has also had headaches and abdominal discomfort; he initially had constipation but has had loose stools for the past several days. Temperature is 40 C (104 F), blood pressure is 114/68 mm Hg, and pulse is 62/min. No oropharyngeal lesions, cervical lymphadenopathy, or neck rigidity is present. Lungs are clear on auscultation, and heart sounds are normal. Abdominal examination shows mild distension and generalized tenderness with hepatosplenomegaly. There is a faint, erythematous, macular skin rash over the trunk and abdomen. Laboratory studies reveal mild, normocytic, normochromic anemia and leukopenia with left shift. Peripheral blood smear is normal. Which of the following is the most likely route of transmission of this patient's infection? A.Bite from a vector insect B.Exposure to a carrier animal C.Ingestion of contaminated food D.Inhalation of infectious droplets E.Sexual contact with infected person

B- salmonella typhi key to figuring out what the answer is: southeast asia, fever reached a plateu, BRADYCARDIA IN THE PRESENCE OF HIGH FEVER, constipation now loose stool (pea soup stools) and rash on the trunke Typhoid fever is a life-threatening infection caused by Salmonella Typhi or Paratyphi. These gram-negative, enteric pathogens reside only in the gastrointestinal tract of humans (Choice B). Transmission occurs via the fecal-oral route, primarily due to ingestion of food or water contaminated by human feces. Therefore, most cases arise in the developing world where sanitation is poor. Travelers who have not been vaccinated (with typhoid vaccine) are at risk and may return clinically ill. Typhoidal Salmonella species invade enterocytes, travel from the apical to the basolateral end of the cell in vacuoles, and emerge in the intestinal submucosa. Here, the organism is phagocytosed by macrophages but is able to evade destruction due to a specialized capsular antigen (Vi) that impairs the macrophage-mediated oxidative burst. The pathogen subsequently replicates in an unchecked fashion within macrophages and spreads through the lymphatic and reticuloendothelial system, leading to widespread, systemic illness. Manifestations typically develop over weeks in a step-wise fashion. Patients usually develop a progressive fever, often with relative bradycardia (pulse-temperature dissociation), followed by abdominal pain, salmon-colored macules on the trunk ("rose spots"), and constipation or diarrhea. Dissemination through the reticuloendothelial system often causes hepatosplenomegaly, anemia, and leukopenia. Later, ulceration of Peyer patches (lymphoid aggregations in the small intestine submucosa) can result in gastrointestinal bleeding or intestinal perforation. Antibiotic treatment is required to prevent complications and death.

A group of researchers is investigating the causative agents responsible for infections in immunocompromised individuals. Patients with solid-organ malignancy who have received conventional chemotherapy are enrolled in the study. Detailed microbiologic data are obtained on any patient who develops fever during chemotherapy-induced neutropenia. Which of the following types of pathogens are most likely to be obtained in the majority of these patients? A) environmental opportunistic infection B) hospital acquired drug resistant C) patients own endogenous D) previously latent reactivated pathogens

C Systemic chemotherapy targets rapidly dividing cancer cells but often incidentally damages other rapidly dividing cells in the gastrointestinal tract and bone marrow, which dramatically increases the risk of infection due to: Mucositis - Damage to rapidly dividing epithelial cells of the digestive tract creates breaches in the mucosal barrier that allow endogenous bacterial flora (commensal organisms) to penetrate and enter the bloodstream. Gram-negative enteric bacilli, such as Pseudomonas aeruginosa, and gram-positive skin organisms, such as Staphylococcus epidermidis and S aureus, are most likely to invade. Neutropenia - Damage to rapidly dividing granulopoietic cells reduces circulating neutrophil count. Because neutrophils serve as front-line defense against bacterial and fungal pathogens, commensal pathogens that penetrate the mucosal/cutaneous barrier cannot be contained and subsequently spread to the bloodstream and deeper tissues. Neutropenic patients have weak inflammatory responses and fever is often the only sign of infection (neutropenic fever). To reduce the risk of serious infection, patients who receive systemic chemotherapy are often empirically treated with prophylactic antibiotics (eg, ciprofloxacin) to prevent commensal organisms from invading and with granulocyte-stimulating agents (eg, filgrastim) to increase neutrophil count.

A 6-year-old boy is brought to the office due to frequent nighttime awakenings. Several times a week over the past month, he has come to his parents' bed around 3 AM, frequently crying, appearing frightened, and saying that he became lost in a crowd and couldn't find his parents. Although he is comforted by their reassurance, he refuses to return to his room and eventually falls back asleep in their bed. The boy is about to start first grade at a new school. He is healthy and developmental milestones are normal. Examination shows no abnormalities. The sleep stage associated with this child's frightening experience is characterized by which of the following? A.Increased duration in first third of the night B.Low-amplitude theta EEG waves C.Muscle paralysis D.Sleep spindles and K complexes on EEG E.Slow-wave sleep (delta EEG waves)

C this child is having nightmares which is when there is recurrent awakening but kids can be comforted an they go back to sleep. It usually happens during REM sleep which is beta waves. muscle paraylisis/motor atonia is common aswell during that sleep as well as dreaming and clitoral and penile tumescence This boy's recurrent awakenings with associated dream content and ability to be comforted are characteristic of nightmares, a common and developmentally normal experience for most children. Nightmares occur during rapid eye movement (REM) sleep and are more frequent in the final third of the night when REM periods become longer (Choice A). Children can often describe the nightmare, become fully alert after awakening, and are typically able to be consoled. In contrast, sleep terrors are a common non-REM parasomnia of childhood that occur during slow-wave sleep in the first third of the night. Unlike nightmares, the child with sleep terrors does not fully awaken, is unresponsive to comfort, and has no associated dream content. REM sleep is characterized by the defining feature of rapid eye movements, vivid dreaming, and voluntary muscle atonia due to inhibition of motor neurons. EEG activity during REM sleep is similar to the waking state, consisting of a combination of alpha, beta, and desynchronous EEG waves (sawtooth waves).

A 24-year-old pregnant woman comes to the emergency department in active labor at full term. The patient has a history of HIV from injection drug use. She is not adherent with antiretroviral therapy and has had no recent testing of CD4 count or plasma viral load. Physical examination shows regular uterine contractions, ruptured amniotic membranes, and a fully dilated cervix. A decision is made to continue with the vaginal delivery due to advanced labor. Intravenous zidovudine is administered to the patient during the delivery and to the infant immediately after birth. This medication helps decrease the risk of perinatal transmission by inhibiting which of the following components of viral genome replication? A.Folate-dependent uracil methylation B.Integration of viral DNA into the host genome C.Nucleoside phosphorylation D.Phosphodiester bond formation E.Proper base pairing and hydrogen bond formation

D HIV is a single-stranded RNA virus that is converted into cDNA by the HIV enzyme reverse transcriptase. This enzyme grows DNA by adding nucleotides to the terminal 3′-hydroxyl group using a 3′-5′ phosphodiester bond. Nucleoside reverse transcriptase inhibitors (NRTIs) such as zidovudine block this step in viral replication. This class of medications is composed of nucleoside/nucleotide analogues that do not have a 3′-hydroxyl group. When reverse transcriptase incorporates an NRTI into a growing DNA strand, it results in chain termination because no 3′-hydroxyl group is available to add additional base pairs. Zidovudine (ZDT) was the first approved HIV medication but is now used primarily to prevent maternal to fetal transmission of HIV during labor and delivery. It is a thymidine analogue that contains an azido group in place of the hydroxyl group usually found at the 3′ end of thymidine.

A 46-year-old woman is hospitalized due to severe depression and fatigue. She has no other medical problems. Routine laboratory tests are ordered and the patient is started on antidepressant medication. After a week, her depression improves slightly and she is discharged home to continue with outpatient care. However, the patient remains severely fatigued and is unable to return to her job, forcing her to take a leave of absence. At her follow-up appointment 2 weeks later, review of the hospital record by the outpatient physician reveals a TSH level of 15.2 µU/mL that was never addressed by the inpatient physician. Subsequent evaluation and treatment with thyroid hormone result in rapid improvement of her depression and fatigue. Which of the following is the appropriate categorization for this type of medical error? A.Malpractice B.Near miss C.Non-preventable adverse event D.Preventable adverse event E.Sentinel event

D Hypothyroidism is a known cause of depression and is routinely screened for with a TSH level. This patient's elevated level (normal: 0.5-5.0 µU/mL) should have been detected during hospitalization and prompted further workup. The inpatient physician's failure to note and address the abnormality is a preventable medical error that resulted in delayed diagnosis. Preventable medical errors involve harm to the patient by an act of commission or omission rather than from the underlying disease and are the result of failure to follow evidence-based best practice guidelines.

A 59-year-old man is being evaluated for markedly elevated prostate-specific antigen levels. He has no bone pain or urinary symptoms. The patient has no chronic medical conditions or surgical history. His father died of prostate cancer. Vital signs are normal. Digital rectal examination reveals an indurated prostate with no palpable nodules. The remainder of the examination is normal. Prostate biopsy is planned. Which of the following is the best way to obtain a diagnosis in this patient? A) Cystoscopy guided; multiple random biopsies of the prostate B) Cystoscopy guided; single biopsy of the central portion of the prostate C) Transperineal approach; fine-needle aspiration of the prostate D) Transrectal route; multiple random biopsies of the prostate E) Transrectal route; single biopsy from the center of the gland

D Most prostate cancer arises in the peripheral zone of the gland, which abuts the rectum. Therefore, prostate biopsies are primarily obtained via the transrectal approach; multiple random core samples of the prostate are typically taken. Because only a small part of the peripheral zone encases the distal urethra, patients with prostate cancer do not typically present with urinary symptoms.

A 3-year-old boy who recently immigrated to the United States is brought to the physician by his parents because he has not yet begun to walk or speak. Assessment of his developmental milestones shows severe intellectual disability. He dies 6 months later from refractory seizures resulting in respiratory failure. Autopsy shows pallor of the substantia nigra, locus ceruleus, and vagal nucleus dorsalis. The underlying condition responsible for this patient's death is most likely caused by a deficiency of which of the following enzymes? A) Branched-chain ketoacid dehydrogenase B) Dopamine hydroxylase C) Homogentisic acid oxidase D) Phenylalanine hydroxylase E) Tyrosinase

D pku. This patient's severe intellectual disability, history of seizures, and abnormal pallor of catecholaminergic brain nuclei on autopsy are suggestive of phenylketonuria (PKU). PKU results from the inability to convert phenylalanine into tyrosine, a reaction which is normally catalyzed by phenylalanine hydroxylase. This enzyme requires the cofactor tetrahydrobiopterin (BH4), which is regenerated from dihydrobiopterin (BH2) by the enzyme dihydropteridine reductase. Although neonatal hyperphenylalaninemia can be caused by deficiency of either enzyme, most cases are attributable to abnormalities in phenylalanine hydroxylase. It is believed that excess phenylalanine and the presence of large concentrations of phenylalanine metabolites (eg, phenyllactate & phenylacetate) contribute to the brain damage seen in PKU. Hypopigmentation involving the skin, hair, eyes, and catecholaminergic brain nuclei (which produce a dark pigment known as neuromelanin) results from the inhibitory effect of excess phenylalanine on melanin synthesis. The classic musty or mousy body odor is due to the accumulation of abnormal phenylalanine metabolites.

A 71-year-old man comes to the office with a 2-month history of progressive exertional dyspnea. The patient is unable to walk half a block without resting and can no longer climb stairs without stopping every few steps. He also has had difficulty sleeping due to repeatedly waking up at night feeling like he is suffocating. Physical examination shows bilateral lower extremity edema and distended neck veins. Cardiac auscultation findings over the apex are given below. This patient's auscultatory findings most likely reflect which of the following? A.Aortic valve sclerosis B.Bulging of the interventricular septum C.Dynamic left ventricular outflow obstruction D.Elevated pulmonary arterial pressure E.Increased left ventricular end-systolic volume

Dialated cardiomyopathy e Cardiac auscultation in this patient reveals a low-frequency sound occurring after S2, known as the third heart sound (S3). Although the S3 can be normal in young individuals and pregnant patients, its presence in those age >40 is typically a sign of left ventricular (LV) volume overload/failure (causing exertional dyspnea and paroxysmal nocturnal dyspnea in this patient). In early diastole, the ventricles relax and the atrioventricular valves open, allowing blood to rush in and fill the ventricles. An S3 develops with forceful rapid passive filling that exceeds the expansion capacity of the ventricle, leading to sudden deceleration of the entering blood column and reverberation of the ventricular walls. As a result, S3 is often heard in pathologic settings causing high ventricular filling pressures and/or volume overload, particularly aortic or mitral regurgitation and dilated cardiomyopathy.

A 65-year-old man comes to the office due to advanced prostate cancer with bone metastases. For the last 4 months, he has been treated with a GnRH agonist, which has significantly reduced his bone pain. However, the patient now reports discomfort at the anterior chest wall. Physical examination reveals bilateral, mildly tender enlargement of the breast tissue behind the nipple-areolar complex. Which of the following medications, if initiated early, could have most effectively prevented this patient's breast symptoms? A. Bicalutamide B. Danazol C. Finasteride D. Spironolactone E. Tamoxifen

E Gynecomastia is the development of glandular breast tissue in males. It is characterized by ductal epithelial hyperplasia with fibrosis of the surrounding stroma. Breast growth is inhibited by androgens and promoted by estrogens; conditions that increase the estrogen to androgen ratio can lead to gynecomastia. Common pathologic causes include cirrhosis (increased aromatase activity, decreased clearance of estrogens), end-stage renal disease (decreased testosterone production), and medications (eg, spironolactone, ketoconazole). Men with prostate cancer who receive androgen-deprivation therapy (eg, orchiectomy, long-acting GnRH agonists, androgen receptor inhibitors) can develop gynecomastia due to dramatic (>95%) reductions in circulating testosterone (with lesser reductions in estrogens). Treatment with tamoxifen, a selective estrogen receptor modulator that acts as an estrogen antagonist in the breast, can reduce the risk of gynecomastia in these patients.

A 64-year-old man comes to the office due to 3 days of diarrhea. He began to have nausea, vomiting, abdominal cramps, and diarrhea 24 hours after eating chicken salad at a fast-food restaurant. The patient continues to have 4-6 episodes of diarrhea per day with no blood in the stool. His wife, who ate the same food, had a few loose stools but felt well afterward. The patient takes a tumor necrosis factor inhibitor for psoriatic arthritis and a proton pump inhibitor for gastritis. Temperature is 38 C (100.4 F), blood pressure is 120/70 mm Hg, and pulse is 88/min. Abdominal examination reveals mild diffuse tenderness to palpation and increased bowel sounds. Cultures of the stool yield gram-negative bacilli that are non-lactose fermenting and oxidase negative. Which of the following is the most likely complication associated with this patient's current infection? A) Chronic malabsorption B) Generalized seizure C)Guillain-Barré syndrome D.) Hemolytic uremic syndrome E.) Long-bone osteomyelitis

E Salmonella Salmonella species are enteric gram-negative bacilli that are oxidase negative and generally non-lactose fermenting. Nontyphoidal strains such as S enterica are a major cause of foodborne gastroenteritis; there are an estimated 2-4 million cases in the United States annually. Transmission typically occurs after the consumption of contaminated poultry products (eg, chicken salad), including eggs. However, handling amphibians and reptiles can also lead to infection. Ingested Salmonella must survive the acidic environment of the stomach prior to traveling to the small intestine to cause disease. Therefore, patients on acid suppression (eg, proton pump inhibitors) require smaller infectious doses and are generally at greater risk. In the small intestine, the organism swims through the mucus barrier, attaches to and penetrates enterocytes, and then travels within vacuoles from the apical end to the basolateral membrane and on to the lamina propria. Here, the pathogen is phagocytosed by macrophages and neutrophils and elicits a profound inflammatory response. Manifestations may be quite mild or may feature fever, nausea, vomiting, and watery diarrhea. Most cases resolve spontaneously within 3-4 days, but a minority of individuals develop invasive disease with spread through the bloodstream and reticuloendothelial system. Long-bone osteomyelitis is a common complication, particularly at sites of prior bone injury/prosthesis or in those with sickle cell disease. Endocarditis, mycotic aneurysm, and visceral abscesses can also occur. The risk of invasion is greatest in patients at the extremes of age and in those with impaired immunity (eg, AIDS, tumor necrosis factor-alpha inhibitor).

A 24-year-old woman comes to the office due to a persistent facial rash. The patient easily develops "sunburns" after sun exposure and her fingers "turn blue" in cold weather. She has also felt more fatigued than usual. Physical examination shows a facial rash in a butterfly distribution that spares the nasolabial folds. Laboratory studies reveal several types of autoantibodies directed against components of the cell nucleus. One specific antibody targets proteins complexed with small nuclear ribonucleic acid. These protein-ribonucleic acid complexes are most likely involved in which of the following cellular functions? A.Aiding mRNA in exiting the nucleus B.Allowing proper functioning of DNA ligase C.Charging tRNA with amino acids D.Polyadenylation of RNA transcripts E.Removal of introns from RNA transcripts F.Synthesizing Okazaki fragments

E This patient's constellation of symptoms (eg, malar rash, photosensitivity, Raynaud phenomenon ["blue fingers"], fatigue) is consistent with systemic lupus erythematosus, an autoimmune disease associated with anti-Smith antibodies (highly specific). Smith protein normally complexes with small nuclear RNA (snRNA) in the cytoplasm, forming small nuclear ribonucleoproteins (snRNPs). Transcription occurs in the nucleus and is catalyzed by 3 types of RNA polymerases, leading to the formation of messenger RNA (mRNA), ribosomal RNA (rRNA), transport RNA (tRNA), and snRNA. RNA polymerase II synthesizes both mRNA and snRNA, the latter of which combines with specific proteins to form snRNPs. mRNA synthesis occurs in 2 stages. During the first, the DNA template is transcribed into a complementary strand of pre-mRNA. In the second, pre-mRNA is processed into mature mRNA through the following steps: RNA capping: Addition of a methylated guanine nucleotide to the 5' end. RNA polyadenylation: Addition of several adenine nucleotides to the 3' end (poly-A tail). RNA splicing: Removal of introns (noncoding regions) by spliceosomes, which consist of snRNPs and other proteins. Mature mRNA then transfers the genetic code to the cytoplasm and serves as a template for protein synthesis (translation).

A research laboratory develops a new serologic test for detecting prostate cancer. The new assay is compared to transrectal ultrasound-guided prostate biopsy to determine sensitivity and specificity parameters. It is found that the test result is negative in 95% of patients who do not have the disease. If the new assay is used on 8 blood samples taken from patients without prostate cancer, what is the probability of all 8 test results coming back negative? a)0.05 * 8 b) 0.95 * 8 c) 0.95^8

Each of the 8 blood sample results is an independent event (ie, one patient's result has no impact on another's) with a 0.95 (95%) probability of correctly testing negative and a 0.05 (5%) probability of incorrectly testing positive. To calculate the chance of all 8 tests testing negative, multiply the probability of each test returning a negative result: 0.95 × 0.95 × 0.95 × 0.95 × 0.95 × 0.95 × 0.95 × 0.95 = 0.958

An 18-year-old woman comes to the office for evaluation of acne. The patient is very upset about the acne scarring, which worsened since she started college. She has been very stressed by upcoming examinations and feels "exhausted" due to lack of adequate sleep. The patient has been using topical benzoyl peroxide and tretinoin for the past year. She was also prescribed an oral antibiotic but stopped taking it several months ago due to lack of improvement. The patient is sexually active with her boyfriend and uses an intrauterine device for contraception. Her periods are regular and last 5 days. She takes no other medications and does not use tobacco, alcohol, or illicit drugs. Physical examination shows nodulocystic acne with scarring on the face, chin, and upper back. The patient is interested in isotretinoin treatment. Which of the following is the best next step in management of this patient? A.Bone density test B.Pelvic ultrasound C.Removal of intrauterine device D.Serum retinol level E.Serum testosterone F.Serum TSH level G.Urine β-hCG

G you dont do pelvic US unless suspecting PCOS Isotretinoin is a synthetic 13-cis-isomer of naturally occurring all-trans-retinoic acid (tretinoin), a derivative of vitamin A. It can be administered orally to treat severe acne associated with significant scarring. It is also indicated in patients who fail therapy with topical tretinoin/benzoyl peroxide and antibiotics. Retinoids inhibit follicular epidermal keratinization, thereby loosening the keratin plugs of comedones and facilitating their expulsion. They also reduce the size of sebaceous glands and inhibit sebum production. Isotretinoin is a potent teratogen and is absolutely contraindicated in pregnancy. Exposure to this medication can cause spontaneous abortion and a variety of severe fetal anomalies. A negative pregnancy test (urine or serum) is required prior to prescribing, and abstinence/contraception is recommended. Monthly pregnancy testing is required during treatment. An intrauterine device (IUD) is a long-term contraception method that is 99% effective and safe to use in women of all ages. It should not be discontinued in this patient (Choice C). Because no contraception method is 100% effective, condom use as a backup method is advised

A 14-year-old girl is brought to the physician for a routine checkup. When asked about her menstrual periods, she says that she does not get them every month. When she does have her period, she bleeds for 7-10 days and needs to use nearly twice as many tampons a day as her older sister. She also complains of occasional spotting that happens outside of her normal periods. The patient underwent menarche a year ago and her last period was 6 weeks ago. She eats a balanced diet and exercises 3 times a week with her volleyball team. Her past medical history is unremarkable, and her body mass index is 25 kg/m2. Which of the following is the most likely cause of this patient's complaints? A) Anovulatory cycles B) Complex atypical hyperplasia of the endometrium C) Endometrial atrophy D) Endometrial stimulation by progesterone E) Increased follicle-stimulating hormone secretion

In most women age 20-40 years, menstrual cycles are consistent and last 24-35 days, with menstrual flow ranging from 4-6 days. However, adolescents typically have an immature hypothalamic-pituitary-ovarian axis for several years following menarche. During this time, they may have longer menstrual cycles and irregular bleeding patterns due to the presence of anovulatory cycles. In the absence of ovulation, the ovarian follicle does not degenerate and become a corpus luteum. As a result, no progesterone is produced and estrogen levels remain persistently high, causing the endometrium to remain in the proliferative phase. Chronically proliferative endometrium becomes disorganized and fragile with unstable venous capillaries, resulting in irregular periods of stromal breakdown with variable, but often heavy, bleeding. By young adulthood, ovulation occurs regularly and menstrual cycles become more predictable. However, as women approach menopause in their late 40s, anovulation becomes more prevalent and menstrual irregularity ensues.

A 30-year-old man comes to the emergency department with severe headache and neck pain following a motor vehicle collision. The patient has no blurry or double vision, dizziness, slurred speech, numbness, or weakness. Neuroimaging is unremarkable except for evidence of a right transverse foramina fracture at the level of C2. The patient is placed in a cervical collar and admitted for further evaluation. While in the hospital, he suddenly develops dizziness, right facial numbness, and hoarseness. Temperature is 36.7 C (98 F), blood pressure is 160/90 mm Hg, pulse is 80/min, and respirations are 12/min. Neurologic examination shows partial ptosis and miosis of the right eye, as well as nystagmus. Hearing is intact bilaterally. There is reduced sensation to pain and temperature on the right side of the face and the left side of the body. The patient is ataxic and displays past-pointing with his right hand on finger-to-nose testing. Vascular imaging will most likely reveal an acute dissection in which of the following arteries? A) Basilar B) Common carotid C) Internal carotid D) Middle cerebral E) Posterior cerebral F) Vertebral

This patient's neurologic findings are consistent with acute ischemia involving the right posterior inferior cerebellar artery (PICA), which arises from the vertebral artery. Occlusion of the PICA results in lateral medullary (Wallenberg) syndrome. Characteristic features include vertigo/nystagmus, ipsilateral cerebellar signs (eg, ataxia), loss of pain/temperature sensation in the ipsilateral face and contralateral body, and bulbar weakness (eg, dysphagia, dysphonia). Patients may also develop ipsilateral Horner syndrome (ie, miosis, ptosis, anhidrosis) due to damage of sympathetic fibers innervating the head. Traumatic vascular dissection is a relatively common cause of ischemic stroke in the young, with most cases affecting the extracranial vessels (eg, carotid and vertebral arteries). The vertebral arteries course through the transverse foramina in the neck before entering the skull at the foramen magnum and therefore can be easily injured by cervical spinal trauma. Progressive dissection can ascend intracranially to involve the section of the vertebral artery where the PICA branches off, leading to delayed development of lateral medullary syndrome (as in this patient with a normal initial examination).

A 29-year-old woman comes to the hospital due to fever and skin rash. She returned from a trip to Brazil 10 days ago. Her symptoms started 5 days ago with a headache, retro-orbital pain, high-grade fever, and joint and muscle pains. She has also had an episode of epistaxis. Today, she noticed a skin rash all over her body. Examination shows a diffuse maculopapular rash and scattered petechiae. Laboratory study results show thrombocytopenia. Her boyfriend traveled to Brazil with her and is asymptomatic. Transmission of the pathogen causing this patient's current symptoms occurs in a manner that is most similar to which of the following diseases? A.Chikungunya fever B.Lymphogranuloma venereum C.Meningococcal meningitis D.Pulmonary tuberculosis E.Rocky Mountain spotted fever F.Typhoid fever

This patient's presentation after recent travel to Brazil is consistent with dengue fever (DF). DF is due to a virus transmitted by the Aedes aegypti mosquito. It is prevalent in tropical and subtropical regions (South and Southeast Asia, Pacific Islands, Caribbean, Americas). DF presents as an acute febrile illness with headache, retro-orbital pain, and joint and muscle pain. Other findings can include hemorrhage (eg, petechiae, purpura, epistaxis, melena), thrombocytopenia, leukopenia, and hemoconcentration. The Aedes mosquito also transmits the virus causing chikungunya, a febrile illness with flulike symptoms, prominent polyarthralgia/arthritis (hands, wrists, ankles), and diffuse macular rash. As a result, many areas have had simultaneous outbreaks of both dengue and chikungunya. Preventive measures against both infections include protective barriers (eg, bed nets, window screens) and insect spraying.

A 72-year-old man with long-standing dyspnea was seen in the clinic after experiencing an episode of syncope. Physical examination showed weak and slowly rising arterial pulses. Cardiac auscultation showed a harsh midsystolic murmur best heard at the second right intercostal space with decreased intensity of the second heart sound. Electrocardiogram and echocardiogram confirmed the diagnosis of severe aortic stenosis. Two months later, the patient comes to the emergency department with palpitations and increased shortness of breath. His blood pressure is 90/60 mm Hg and his heart rate is 130/min with an irregularly irregular rhythm. Electrocardiogram shows new-onset atrial fibrillation without significant ST-segment or T-wave changes. Chest x-ray shows bilateral pulmonary edema. Which of the following hemodynamic changes is most likely associated with this patient's current presentation?

This patient's sudden-onset heart failure was most likely precipitated by acute atrial fibrillation, a condition that occurs in up to 10% of patients with severe aortic stenosis (AS). Patients with severe AS already have reduced cardiac output due to significant valvular obstruction, which can be exacerbated by the sudden loss of normal atrial contraction that contributes to ventricular filling. Atrial contraction is especially important for these patients as many have concentric left ventricular (LV) hypertrophy and therefore reduced LV compliance. As a result, they become dependent on atrial contraction to maintain adequate LV filling. Without atrial contraction, LV preload can decrease to the point of producing severe hypotension. In addition, loss of the atrial kick can result in significantly increased mean pulmonary venous pressure due to buildup of blood in the left atrium and pulmonary veins, leading to acute pulmonary edema. As a result, cardioversion is indicated for acute atrial fibrillation in patients with severe AS.

A 10-year-old girl is brought to the office for evaluation of short stature. She was an average-sized infant, but over the past few years, her height growth velocity has plateaued. The patient has not menstruated and has no symptoms. She takes no medications, has no allergies, and has received all recommended immunizations. Menarche occurred in her mother at age 14 and both of her parents are tall. The patient's height is at the <5th percentile and weight is at the 50th percentile for age and sex. She has no breast buds and no axillary or pubic hair. She has a low hairline, a short and wide neck, a broad chest, and widely spaced nipples. Which of the following is the most likely underlying mechanism for this patient's condition? A. Balanced translocation B. Frameshift mutation C. Meiotic nondisjunction D. Trinucleotide repeat expansion E. Uniparental disomy

Turner syndrome is a genetically heterogeneous condition that is most commonly due to meiotic nondisjunction during gametogenesis. The loss of one X chromosome in the sperm or egg results in a missing X chromosome in all of the patient's cells (45,X). In other patients, the nondisjunction event occurs during mitosis in early embryogenesis; these patients are missing the X chromosome in only some of their cells (mosaic Turner syndrome [45,X/46,XX]). A minority of patients have both X chromosomes, but one is structurally abnormal and missing some genetic material (eg, X fragments, isochromosomes). The loss of all or part of the X chromosome in Turner syndrome results in a missing SHOX gene, which normally promotes long bone growth. Therefore, patients with this syndrome typically have short stature. Meiotic nondisjunction is also responsible for Klinefelter syndrome and trisomies 13, 18, and 21.

A prospective study evaluates the relationship between regular antioxidant supplement use (vitamins C and E) and the risk of stroke in healthy and physically active men age 40-60. The study compares the risk of stroke among men who consumed antioxidant supplements for ≥5 years and among men who consumed antioxidant supplements for <5 years, as compared to a reference group of men who never consumed antioxidant supplements. According to the study results, men who consumed antioxidant supplements for <5 years and men who consumed antioxidant supplements for ≥5 years have stroke relative risks of 0.95 (p = 0.45) and 0.75 (p < 0.01), respectively, when compared to the reference group. The results of the study were adjusted to account for baseline differences related to healthy behaviors and overall health. Which of the following factors most likely explains why the relative risk of stroke is lower with longer antioxidant use?

accumalation effect Antioxidant use for ≥5 years was associated with a statistically significant (p < 0.01) decreased risk of stroke (relative risk [RR] = 0.75 <1). By contrast, antioxidant use for <5 years demonstrated a nonstatistically significant (p > 0.05) smaller reduction in risk (RR = 0.95); in other words, antioxidant use for <5 years seems to have no benefit on stroke risk reduction. This discrepancy can be explained if reduction in stroke risk is associated with the duration of exposure to antioxidant supplementation; in other words, a minimum cumulative exposure to antioxidants may be necessary for the exposure to have a significant effect on stroke risk. This concept of accumulation effect can apply to both risk factors and risk reducers. The effect of exposure to risk factors may depend on the duration and intensity of the exposure; long-term exposure may be necessary well before an effect on the disease process is clinically evident (eg, lung cancer developing after decades of smoking exposure). Similarly, exposure to certain risk reducers must occur continuously over extended periods before disease outcome is affected. In this case, ≥5 years of continuous antioxidant use (risk reducer) were required to reveal their protective effect on stroke.

A previously healthy 15-year-old girl is brought to the emergency department due to respiratory distress and continuous cough after eating at a buffet in a local restaurant. Blood pressure is 80/40 mm Hg, pulse is 130/min, and respirations are 32/min. She is in marked respiratory distress but is able to speak. Diffuse wheezing and erythematous skin rash are present. Which of the following hemodynamic changes are most likely present in this patient? CVP, PCWP, and SVR?

all decreased but remember cardiac out put increases in early stages This patient with respiratory distress, wheezing, and erythematous skin rash following food exposure likely has anaphylaxis; her severe hypotension and tachycardia indicate anaphylactic shock. The primary disturbance in distributive shock (eg, septic shock, anaphylactic shock) is profound peripheral vasodilation which, in anaphylactic shock, results from widespread IgE-mediated release of inflammatory mediators (eg, histamine). This has the following effects: Arteriolar vasodilation causes decreased systemic vascular resistance (SVR) and consequent hypotension. Venular vasodilation causes decreased central venous pressure (CVP) and reduces venous return to the right atrium (Choice D). The vasculature becomes leaky, resulting in loss of intravascular volume that further contributes to both hypotension and decreased CVP. With reduced venous return to the right atrium, there is reduced right ventricular output and reduced blood delivery to the left atrium and left ventricle. This leads to decreased pulmonary capillary wedge pressure (PCWP), a reflection of left atrial pressure. Despite the reduced left ventricular preload, cardiac output is commonly normal or increased in early distributive shock due to both the large decrease in SVR and a baroreceptor reflex-mediated increase in heart rate (ie, tachycardia). The increased cardiac output leads to increased mixed venous oxygen saturation due to rapid blood transit through the peripheral capillaries and incomplete oxygen uptake by the tissues.

An intern in the endocrinology department is asked to complete a chart review of cholesterol levels of all patients hospitalized with diabetes mellitus-related cardiovascular complications in the past 6 months. The intern's preliminary data analysis reveals that there were 400 such patients, and that serum cholesterol levels were normally distributed among the patients with a mean of 220 mg/dL and a standard deviation of 10 mg/dL. Based on these results, how many patients in this study would be expected to have serum cholesterol >240 mg/dL? A.2 B.10 C.20 D.64 E.128

b

A 28-year-old woman, gravida 3 para 3, comes to the emergency department with severe abdominal pain in the left lower quadrant and vaginal bleeding. She is saturating a pad every 3-4 hours. Her last menstrual period was 6 weeks ago. Past surgical history is significant for 3 cesarean deliveries and permanent sterilization via a bilateral tubal ligation. A urine pregnancy test is positive, and an ultrasound shows a 2-cm mass in the left adnexa adjacent to the ovary and a thickened endometrial stripe. If a uterine curettage is performed, which of the following findings would be expected in this patient? A) atypical endometrial cells that form glands. B)dilated, coiled endometrial glands and vascularized edematous stroma. C) Enlarged chorionic villi and avascular edematous stroma. D) An inflammatory endometrial infiltrate E) Straight, short endometrial glands and compact stroma.

b Ectopic pregnancy occurs when a fertilized ovum implants outside of the uterus. The most common site is the ampulla of the fallopian tube, which may appear as an adnexal mass on ultrasound. Risk factors include tubal pathology, such as from previous infection or surgery (eg, tubal ligation). Pregnancy after permanent sterilization is extremely rare, but one-third of cases are ectopic if implantation occurs. An ectopic pregnancy may become life-threatening as the embryo and trophoblastic tissue proliferate. This growth will compromise the blood supply to the surrounding tissues, which can result in rupture and profuse intra-abdominal bleeding. A ruptured ectopic pregnancy is managed surgically by removing the pregnancy and achieving hemostasis. Dilation and curettage of the uterus may be performed either to stop uterine bleeding or confirm whether the pregnancy is intrauterine or ectopic. In an ectopic pregnancy, the uterine specimen would reveal decidualized endometrium only, consistent with dilated, coiled endometrial glands and vascularized edematous stroma. These changes occur in the luteal phase of the menstrual cycle, under the influence of progesterone, as the endometrium prepares for implantation. Embryonic and trophoblastic tissue will be absent from the uterus. atypical endometrial cells that form glands would suggest endometrial adenocarcinoma, a malignancy that typically occurs in postmenopausal women and manifests with vaginal bleeding. A molar pregnancy or spontaneous abortion may present with vaginal bleeding. Uterine curettage may show enlarged chorionic villi and avascular edematous stroma. In contrast, intrauterine chorionic villi are absent in an ectopic pregnancy. An inflammatory endometrial infiltrate would suggest endometritis, an infection of the decidua, which presents with uterine tenderness, fever, and tachycardia. An ectopic pregnancy is not an infectious process. Straight, short endometrial glands and compact stroma are found in the early proliferative phase of the menstrual cycle. This microscopic appearance would be observed 4-7 days after the onset of menses.

An 82-year-old man is admitted to the hospital in the middle of the night. He is non-communicative and has multiple comorbidities, including end-stage lung cancer. The patient has not designated a specific power of attorney and does not have an advance directive on file. His wife of many years, who is at the hospital with him, has an extensive conversation with the physician in charge of his care. During the conversation, she says that the patient had mentioned on several occasions not to resuscitate him if he were near death as he did not want to live "attached to machines or brain dead." She says that he just wants to die in peace when the time comes. What would be the most appropriate course of action if the patient enters ventricular fibrillation? A) place defib pads and administer vasopressin only B) inform nursing staff that the patient is DNR and suspend all treatment C) Attend to patients comfort and be allow for the family to be there as the patient is dying.

c

A 4-week-old boy is hospitalized with persistent vomiting, fussiness, and feeding intolerance. The emesis was initially clear but became bilious in the last few hours. The infant had been breastfeeding exclusively. His temperature is 37.2 C (99.0 F). Blood pressure and pulse are normal. Physical examination shows a normal abdomen with no rebound or guarding. An upper gastrointestinal series is obtained urgently to evaluate for malrotation and volvulus and shows normal rotation but constriction of the duodenum. An abdominal CT scan reveals pancreatic tissue encircling the duodenum. Which of the following is the most likely cause of this patient's condition? A)Abberant differentiation of the midgut structure B)failure of ventral and dorsal pancreatic bud to fuse C) abnormal migration of the ventral pancreatic bud d) failure of apoptosis in the dorsal pancreatic bud

c This patient has acute pancreatitis and radiographic evidence of annular pancreas, which results from abnormal migration of the ventral pancreatic bud. This bud is a foregut derivative that appears by the fifth week of gestation and rotates behind the duodenum during the seventh week of fetal development. The ventral bud moves toward the midline, where it fuses with the dorsal pancreatic bud during the eighth week. The ventral pancreatic bud gives rise to the uncinate process portion of the head and the main pancreatic duct (of Wirsung). The pancreatic tail, body, and the remainder of head derive from the dorsal bud. Abnormal migration of the ventral pancreatic bud can be caused by adherence to either the dorsal bud or duodenum before the rotation begins. This abnormal migration leads to annular pancreas, a rare congenital anomaly in which pancreatic tissue completely surrounds the second part of the duodenum. Annular pancreas can compress the duodenal lumen (causing obstruction) or result in obstructed pancreatic drainage (acute or chronic pancreatitis). However, the majority of patients with annular pancreas are asymptomatic.

A 42-year-old man with HIV comes to the emergency department with 3 days of abdominal pain, decreased appetite, and diarrhea. Temperature is 38 C (100.4 F), blood pressure is 98/60 mm Hg, and pulse is 110/min. Abdominal examination shows tenderness over the left lower quadrant without guarding or rebound tenderness. CD4 count is 38/mm3. Colonoscopy shows erythematous, friable mucosa with superficial ulcerations in the sigmoid and descending colon. Biopsy from the ulcers reveals inflammatory infiltrate, mucosal necrosis, and enlarged endothelial cells with large, ovoid nuclei containing prominent basophilic deposits. Item 1 of 2 Which of the following is the most likely cause of this patient's gastrointestinal symptoms? A.Disseminated mycobacterial infection B.Infection by a modified acid-fast protozoan C.Mucosal invasion by a commensal yeast D.Reactivation of a herpesvirus

d Cytomegalovirus (CMV) is the most common viral opportunistic infection of the gastrointestinal tract in patients with advanced AIDS. Initial infection with CMV is usually asymptomatic or similar to Epstein-Barr virus-induced mononucleosis. As with all members of the herpesvirus family, latent infection is established after resolution of the initial infection. Reactivation can occur at any time but is most common in the setting of immunosuppression. In patients with HIV, CMV colitis is the second most common reactivation syndrome after CMV retinitis. Patients with CMV colitis have fever, anorexia, weight loss, and abdominal pain. The pain may be vague initially but becomes more localized as the disease progresses. Colonoscopy shows mucosal erythema, erosions, and ulcerations. Histologic examination of biopsy specimens will reveal acute and chronic inflammatory changes, vasculitis, and giant cells with large, ovoid nuclei containing centralized intranuclear basophilic inclusions (owl's eye appearance).

A 45-year-old woman comes to the office due to polyuria and nocturia. She has no fever, dysuria, or abdominal pain. The patient has no significant medical problems and takes no medications. Her temperature is 36.7 C (98 F), blood pressure is 120/80 mm Hg, and pulse is 76/min. The patient's mucous membranes appear dry. The remainder of her physical examination is normal. Her urine output and osmolality remain unchanged with water deprivation for several hours, but after administration of desmopressin, urine output decreases and urine osmolality increases. Renal clearance of which of the following substances would decrease the most after this patient's injection? A) Calcium B)Creatinine C)Glucose D)urea

d Polyuria that resolves with the administration of desmopressin (DDAVP, synthetic analogue of vasopressin) is likely secondary to deficient vasopressin secretion (central diabetes insipidus). Vasopressin produces a V2 receptor-mediated increase in water permeability within the cortical and medullary collecting ducts. As water leaves the tubular fluid, urea concentration greatly increases in these tubular segments. Although the cortical collecting duct is impermeable to urea, vasopressin activates urea transporters in the medullary collecting duct, increasing urea reabsorption and decreasing renal urea clearance. This passive reabsorption of urea into the medullary interstitium in the presence of ADH significantly increases the medullary osmotic gradient, allowing the production of maximally concentrated urine.

A 43-year-old woman comes to the office due to increasing fatigue and weight gain despite decreased food intake. She states, "I have not been feeling like myself lately." The patient asks to be referred to a dermatologist as she has always had "nice" skin that has now become dry. Her hair has been thinning and she hopes that a dermatologist can help with that too. Laboratory evaluation shows high serum TSH, low triiodothyronine (T3), and low thyroxine (T4) levels. The patient has a family member with fatigue whose energy level increased after liothyronine (T3) supplementation, and she asks to try this medication. Administering this therapy would most likely result in which of the following hormone level changes in this patient? T3, T4, TSH and reverse T3?

exogenous thyroid hormone is T3 so thatll increase which will cause TSH levels to decrease leading to decrease in T4 and rT3 TSH from the anterior pituitary stimulates the thyroid to produce thyroxine (T4) and a small amount of triiodothyronine (T3). T4 is converted in peripheral tissues to T3 (active form) and reverse T3 (inactive form). TSH secretion is under negative feedback by thyroid hormone on the hypothalamus and pituitary

A 27-year-old man is brought to a family therapist by his wife following a violent outburst in which he nearly injured her. They were having what seemed like a minor argument over a miscommunication about her being late when he suddenly flew into a rage, started shouting, and threw several plates against the wall. His wife is now threatening to leave him because similar episodes keep happening despite his promise to control his anger. The patient is remorseful and says, "I have been getting into trouble because of my temper since high school. Once I get angry, I feel out of control and it's impossible to stop." The patient has no medical history. He drinks beer and uses cannabis to relax approximately 2-3 times a month. Which of the following is the most likely diagnosis in this patient? A) Antisocial personality disorder B) Bipolar disorder, manic episode C) Borderline personality disorder D)Conduct disorder E) Disruptive mood dysregulation disorder(24%) F) Intermittent explosive disorder(52%) G) Phencyclidine intoxication

f This patient's repetitive outbursts of impulsive aggression that are grossly disproportionate to the situation is suggestive of intermittent explosive disorder (IED), an impulse control disorder. Patients with IED often feel a sense of heighted arousal or tension that quickly escalates to rage and uncontrollable impulses to be verbally or physically aggressive. Outbursts may provide an immediate sense of relief that is usually followed by remorse. Patients can be assaultive and destroy property, resulting in significant functional impairment due to interpersonal difficulties, school suspension, job loss, and legal problems. IED is not diagnosed if the violent behavior is premeditated or better explained as a manifestation of another disorder, such as a manic episode of bipolar disorder or substance intoxication. Treatment consists of cognitive-behavioral therapy and pharmacotherapy with selective serotonin reuptake inhibitors.


Ensembles d'études connexes

Ch 8-1 Vocabulary 1960's America Pages 845-894

View Set